Электронная библиотека
Форум - Здоровый образ жизни
Акупунктура, Аюрведа Ароматерапия и эфирные масла,
Консультации специалистов:
Рэйки; Гомеопатия; Народная медицина; Йога; Лекарственные травы; Нетрадиционная медицина; Дыхательные практики; Гороскоп; Правильное питание Эзотерика


Об авторах

Альфред Позаментье — в настоящее время декан педагогического факультета и профессор математического образования в колледже Мерси, Нью-Йорк, в прошлом почетный преподаватель Городского технологического колледжа Городского университета Нью-Йорка. Он почетный профессор математического образования Городского университета Нью-Йорка и бывший декан педагогического факультета, где проработал 40 лет. Позаментье автор и соавтор более 55 книг по математике для преподавателей, учащихся средних и начальных школ, а также широкого круга читателей. Статьи д-ра Позаментье по вопросам образования можно часто встретить в газетах и журналах.

После получения степени бакалавра в области математики в Хантер-колледже Городского университета Нью-Йорка он получил должность преподавателя математики в средней школе Теодора Рузвельта (в Бронксе, Нью-Йорк), где сосредоточился на развитии навыков учащихся, связанных с решением задач, и одновременно на расширении традиционной программы обучения. За шесть лет работы благодаря его усилиям в школе были созданы математические группы (младшего и старшего уровней). Позаментье продолжает работать с учителями математики и методистами, помогая им повышать эффективность преподавания.

Сразу после прихода на математический факультет Городского колледжа в 1970 г. (после получения там степени магистра в 1966 г.) он занялся созданием курса повышения квалификации без отрыва работы для учителей математики средних школ, включавшего такие специальные области, как занимательная математика и решение задач в математике. Как декан педагогического факультета Городского колледжа, Позаментье на протяжении 10 лет всесторонне занимался вопросами образования. За это время ему удалось поднять рейтинг факультета в штате Нью-Йорк до высшего уровня с идеальной оценкой Национального совета по аккредитации подготовки учителей (NCATE) в 2009 г.

В 2014 г. д-р Позаментье повторил свое достижение, подняв педагогический факультет Колледжа Мерси до статуса единственного в США, получившего идеальную первоначальную аккредитацию одновременно и в NCATE, и в Совете по аккредитации подготовки преподавателей (CAEP).

В 1973 г. д-р Позаментье получил степень доктора философии в Фордэмском университете (Нью-Йорк) в области математического образования, и с той поры заслужил профессиональное уважение не только в США, но и в Европе. Он является приглашенным профессором университетов в Австрии, Англии, Германии, Чешской Республике и Польше, а в Венском университете — профессором по программе Фулбрайта (1990 г.).

В 1989 г. д-р Позаментье был удостоен звания «почетный член» в Университете Саут-Бэнк (Лондон, Англия). В знак признания его достижений в области преподавания Ассоциация выпускников Городского колледжа называла его «преподавателем года» в 1994 и 2009 гг. В Нью-Йорке в его честь был устроен праздник 1 мая 1994 г. В 1994 г. он получил «Большую медаль почета» от Республики Австрия, а в 1999 г. с одобрения парламента президент Австрии присвоил ему звание «Профессор университета Австрии». В 2003 г. он получил звание Ehrenb?rger (Почетный член) Венского технологического университета, а в 2004 г. — Почетный крест за достижения в области искусства и науки первого класса. В 2005 г. его имя увековечили и Зале славы выпускников Хантер-колледжа, а в 2006 г. Ассоциация выпускников Городского колледжа вручила ему престижную медаль Таунсенда Харриса. Его имя с 2009 г. присутствует в Зале славы преподавателей математики штата Нью-Йорк, в 2010 г. он получил в Берлине почетную премию Кристиана Петера Бойта.

Д-р Позаментье занимает целый ряд руководящих должностей в сфере преподавания математики на местном уровне. Он является членом Полномочной комиссии при комиссаре по вопросам образования в штате Нью-Йорк и Комитета по стандартам математического образования, а также входит в консультативный совет канцлеров школ г. Нью-Йорк.

Д-р Позаментье — один из ведущих обозревателей по вопросам образования и продолжает активно искать пути повышения интереса к математике как у учителей и учащихся, так и у широкой публики. Это ясно видно по его книгам: Numbers: Their Tales, Types and Treasures (Prometheus, 2015), Teaching Secondary Mathematics: Techniques and Enrichment Units, 9th Ed. (Pearson, 2015), Mathematical Curiosities: A Treasure Trove of Unexpected Entertainments (Prometheus, 2014), Geometry: Its Elements and Structure (Dover, 2014), Magnificent Mistakes in Mathematics (Prometheus Books, 2013), 100 Commonly Asked Questions in Math Class: Answers that Promote Mathematical Understanding, Grades 6–12 (Corwin, 2013), What successful Math Teacher Do: Grades 6–12 (Corwin, 2006, 2013), The Secrets of Triangles: A Mathematical Journey (Prometheus Books, 2012), The Glorious Golden Ratio (Prometheus Books, 2012), The Art of Motivating Students for Mathematics Instruction (McGraw-Hill, 2011), The Pythagorean Theorem: Its Power and Glory (Prometheus, 2010), Mathematical Amazements and Surprises: Fascinating Figures and Noteworthy Numbers (Prometheus, 2009), Problem Solving in Mathematics: Grades 3–6: Powerful Strategies to Deepen Understanding (Corwin, 2009), Problem-Solving Strategies for Efficient and Elegant Solutions, Grades 6–12 (Corwin, 2008), The Fabulous Fibonacci Numbers (Prometheus Books, 2007), Progress in Mathematics, K-9 textbook series (Sadlier-Oxford, 2006–2009), What Successful Math Teacher Do: Grades K-5 (Corwin, 2007), Exemplary Practices for Secondary Math Teachers (ASCD, 2007), 101+ Great Ideas to Introduce Key Concepts in Mathematics (Corwin, 2006), ?, A Biography of the World's Most Mysterious Number (Prometheus Books, 2004), Math Wonders: To Inspire Teachers and Students (ASCD, 2003), и Math Charmers: Tantalizing Tidbits for the Mind (Prometheus Books, 2003).


Стивен Крулик — почетный профессор математического образования в Университете Темпл в Филадельфии. В университете д-р Крулик отвечает за преддипломную и последипломную подготовку преподавателей математики для 1–12 классов, а также за повышение квалификации преподавателей математики. Он читает разнообразные курсы, в числе которых история математики, методы преподавания математики и обучение методам решения задач. Последний курс появился как результат интереса к решению задач и логическому рассуждению на уроках математики. Основой этого интереса стало его стремление к тому, чтобы учащиеся понимали красоту и ценность решения задач, а также умели логически рассуждать.

Д-р Крулик получил степень бакалавра в области математики в Бруклинском колледже Городского университета Нью-Йорка, а степень магистра и степень доктора педагогических наук в области математики — в Педагогическом колледже Колумбийского университета. До прихода в Университет Темпл он в течение 15 лет преподавал математику в школах г. Нью-Йорк. В средней школе Лафайетт в Бруклине он организовал ряд курсов для подготовки учащихся к SAT-экзамену с акцентом на освоении методов решения задач вместо механического запоминания алгоритмов.

На общенациональном уровне д-р Крулик входит в состав комитета Национального совета преподавателей математики, отвечающего за разработку Профессиональных стандартов преподавания математики. В 1980 г. он был редактором ежегодника Национального совета под названием «Решение математических задач в школе». На региональном уровне он был президентом Ассоциации преподавателей математики Нью-Джерси, в 1993 г. входил в редакционный совет математического справочника The New Jersey Calculator Handbook и в 1997 г. редактировал монографию Tomorrow's Lessons.

Основная сфера его интереса — обучение методам решения задач и умению логически рассуждать, методические материалы по преподаванию математики, также комплексная оценка математических способностей. Он является автором и соавтором более 30 книг для преподавателей математики, в том числе Roads to Reasoning (классы 1–8) и Problem Driven Math (классы 3–8). Д-р Крулик, помимо прочего, является основным автором задач для серии базовых учебников. Д-р Крулик часто выступает со статьями по математическому образованию в профессиональных журналах. Он консультировал и проводил семинары в школьных округах США и Канады, а также выступал с лекциями в Вене (Австрия), Будапеште (Венгрия), Аделаиде (Австралия) и Сан-Хуане (Пуэрто-Рико). Д-ра Крулика часто приглашают на национальные и международные совещания, где он концентрирует внимание на формировании у всех учащихся умения логически рассуждать и решать задачи как на уроках математики, так и в жизни.

В 2007 г. в Университете Темпл ему вручили премию «Выдающийся учитель». В 2011 г. Национальный совет преподавателей математики представил его к почетной премии за «Выдающиеся заслуги в сфере математического образования».

Введение

С начала 1980-х гг. решение задач, логическая аргументация и критическое мышление стали неотъемлемой частью программы по математике в США, а потом и в большей части мира. Еще в 1977 г. Национальный совет учителей математики заявил, «обучение решению задач — это главная причина изучения математики». В конце концов, что толку от понимания, как делать то или другое, если не знаешь, когда делать это. Движение под флагом приобретения навыков решения задач набирает силу и распространяется все больше на программу изучения математики. По мере углубления этого процесса он начинает переключаться и на решение задач в повседневной жизни. Каждый день люди сталкиваются с задачами, требующими решения. Они могут варьировать от очень простых, например, что надеть сегодня, до очень сложных. Даже то, что кажется простым, взять хотя бы переход улицы, может оказаться сложным и потребовать обдумывания, если мы приезжаем в страну, где уличное движение организовано иначе.

Прежде чем говорить о решении задач, нужно определиться с тем, что составляет задачу. Задача — это ситуация, в которой необходимо принять решение, но путь к этому решению заранее неизвестен. Запомните эти слова: «но путь к этому решению заранее неизвестен». Когда многие из нас ходили в школу, задачи, которые нас учили решать, нередко были «типовыми». Иначе говоря, «задачи, связанные с возрастом» решались одним путем, «задачи на движение» — другим, а еще были «задачи на смешивание», «задачи, связанные с измерением объема жидкости» и т. д., которые решались своими способами. Фактически, как только мы осваивали определенный метод, соответствующие задачи переставали быть задачами, требующими решения. Все, что нужно было сделать, это определить тип задачи и применить подходящий автоматический процесс.

История математических достижений полна прорывов, реакция на которые нередко выражается словами «мне и в голову не приходило, что можно использовать такой подход». Даже сегодня, когда представляют хорошее или изящное решение задачи, многие реагируют именно так. Решение задач — это попытка сделать такие необычные решения частью досягаемой базы знаний.

Решение задач сегодня в значительной мере основывается на эвристической модели, описанной Джорджем Пойа в книге «Как решить задачу» (How to Solve It), которая была издана в 1945 г. и до сих пор пользуется спросом. В этой книге Пойя представил такой четырехэтапный план решения задач:

1. Уяснение сути задачи.

2. Составление плана.

3. Выполнение плана.

4. Оценка найденного решения.


Большинство нынешних моделей решения задач строятся именно на этой четырехэтапной эвристической модели. План обычно включает в себя: 1) чтение условий задачи; 2) выбор подходящей стратегии, 3) решение задачи и 4) оценка найденного решения или его осмысление. Ключевым аспектом всего процесса является выбор подходящей стратегии, или определение подхода к задаче. Наша книга посвящена детальному исследованию именно этого критически важного этапа.

Итак, выбор подходящей стратегии является ключевым аспектом решения задачи. За последние десятилетия разные авторы описали и представили множество стратегий. В основе большинства из них лежат одни и те же идеи. В этой книге рассматриваются 10 наиболее ценных, на наш взгляд, стратегий решения задач. Каждой из них посвящена отдельная глава. При представлении задачи мы пытаемся сначала предположить, каким будет наиболее очевидный или распространенный подход. Чаще всего он приводит к правильному ответу. Вместе с тем самый употребительный подход нередко требует довольно запутанного математического аппарата, сложных вычислений, а в некоторых случаях дает неправильный ответ.

Затем мы предлагаем более изящное, или образцовое решение, показывающее, как рассматриваемая стратегия решения задачи приводит к ответу. Обратите внимание на то, что мы разделяем «ответ» и «решение». Решение — это процесс от момента чтения условий задачи до момента получения окончательного ответа и его осмысления. Некоторые говорят, что конкретный ответ — это всего лишь одна из наименее важных частей решения. Да, должно быть, так и есть, но процесс, в результате которого получается ответ, является критической частью решения.

По мере того, как вы будете читать эту книгу (и, мы надеемся, прорабатывать предложенные задачи), учитывайте, что во многих случаях для решения задачи можно использовать несколько стратегий. Например, решение задачи с применением стратегии «обоснованное предположение и проверка» обычно требует организации данных в определенном порядке. Когда такое происходит, мы переносим задачу в более подходящую, на наш взгляд, главу.

Каждую главу в этой книге мы начинаем с описания конкретной стратегии, показывающего, как ее можно использовать в каждодневных ситуациях, а затем приводим примеры применения в математике. После этого мы представляем ряд задач, которые лучше всего решаются с помощью именно этой стратегии. Каждая задача — это попытка проиллюстрировать применение конкретной стратегии. В число стратегий, которые мы собираемся рассмотреть, входят:

1. Логическое рассуждение.

2. Распознавание закономерности.

3. Действие от обратного.

4. Принятие другой точки зрения.

5. Анализ экстремальных ситуаций.

6. Решение более простой аналогичной задачи.

7. Организация данных.

8. Схематичное изображение, или визуальное представление.

9. Учет всех возможностей.

10. Обоснованное предположение и проверка.


Как мы уже говорили, редко когда задачу можно решить единственным способом. Решение, которое мы демонстрируем, представляет собой всего лишь один иллюстративный пример. Мы предлагаем читателю попытаться найти другие решения, возможно, более интересные и необычные. Если это вам удастся, мы скажем, что вы молодец! Кроме того, в некоторых случаях, когда доступно несколько стратегий, можно с разным успехом использовать их сочетания.

Чтобы показать, как можно подойти к задаче (и решить ее) с использованием различных стратегий, мы обычно даем несколько решений.

Задача

В комнате, где находятся 10 человек, все поздоровались друг с другом, однократно пожав руку. Сколько всего было рукопожатий?

Решение 1

Воспользуемся стратегией визуального представления и построим схему. В ней 10 точек (которые расположены так, что никакие три из них не находятся на одной прямой), представляющих 10 людей. Начнем с человека, представленного точкой А.



Мы соединяем точку А с каждой из остальных девяти точек и, таким образом, обозначаем первые девять рукопожатий.

Далее, из точки B исходят восемь дополнительных рукопожатий (поскольку А уже поздоровался с B, и линия AB уже построена). Аналогичным образом из точки C можно провести только семь линий к другим точкам (линии AC и BC уже построены), из точки D — шесть дополнительных линий и т. д. Когда мы дойдем до точки I, останется только одно доступное рукопожатие, а именно I с J, поскольку I уже поздоровался с A, B, C, D, E, F, G и H. Таким образом, сумма рукопожатий составит 9 + 8 + 7 + 6 + 5 + 4 + 3 + 2 + 1 = 45. Это то же самое, что получается при использовании формулы для суммы первых n натуральных чисел: где n >= 2. (Обратите внимание на то, что последний рисунок — это десятиугольник, у которого построены все диагонали.)


Решение 2

Для решения задачи можно использовать стратегию учета всех возможностей. Возьмем показанную ниже сетку, в которую включены 10 человек, A, B, C, …, H, I, J, пожимающие друг другу руки. Диагональ с символами X показывает, что люди не могут пожимать руки самим себе.

Оставшиеся клетки показывают двойное число всех других рукопожатий (т. е. A пожимает руку B, а B пожимает руку A). Таким образом, нам нужно взять общее количество клеток (102), вычесть из него количество клеток на диагонали (10) и разделить результат на два. В результате мы получаем:



В общем случае для сетки размером n x n результат будет равен что эквивалентно формуле приведенной выше.

Решение 3

Попробуем теперь решить задачу с помощью принятия другой точки зрения. Возьмем комнату, где находятся 10 человек, каждый из которых пожимает руку остальным девяти. Можно предположить, что число рукопожатий будет равным 10 x 9, или 90. Однако нам нужно разделить это число на два, чтобы устранить дублирование (поскольку рукопожатие A с B можно рассматривать как рукопожатие B с A), и мы получаем

Решение 4

Теперь подойдем к решению задачи через распознавание закономерности. В таблице, представленной ниже, мы перечисляем количество рукопожатий в комнате по мере увеличения числа присутствующих.



В третьей колонке, где приведено суммарное количество рукопожатий, представлена последовательность чисел, называемых треугольными, разность между которыми возрастает каждый раз на единицу. Таким образом, можно просто заполнять таблицу до тех пор, пока мы не достигнем суммы, соответствующей 10 человекам. Можно заметить следующую закономерность: результат в каждой строке равен половине произведения количества людей в этой строке на количество людей в предыдущей строке.

Решение 5

Посмотрим теперь, как задача решается с помощью стратегии организации данных. В таблице, представленной ниже, показан номер человека, входящего в комнату, и количество рукопожатий, которыми он обменивается, с учетом того, что присутствующие уже поздоровались друг с другом, а вошедший не пожимает руку сам себе. Итак, человек номер 10 пожимает руку девятерым, человек номер 9 пожимает руку восьмерым и т. д. Наконец, мы доходим до человека номер 2, который пожимает руку только одному, и человека номер 1, которому здороваться не с кем. И вновь мы получаем сумму, равную 45.


Решение 6

Можно также объединить решение более простой задачи с визуальным представлением (схематичным изображением), организацией данных и распознаванием закономерности. Начнем с рассмотрения одного человека, представленного одной точкой. Здесь, очевидно, мы имеем ноль рукопожатий. Затем увеличим количество людей до двух, представленных двумя точками. В этом случае у нас будет одно рукопожатие. Увеличим количество людей до трех. Теперь получим три рукопожатия. Продолжим увеличивать количество людей до четырех, пяти и т. д.



Задача становится геометрической, где ответом является количество сторон и диагоналей «n-угольника». Таким образом, для 10 человек мы получаем 10-угольник, у которого число сторон n = 10. Для определения количества диагоналей можно использовать формулу:



Итак, количество рукопожатий = 10 + 35 = 45.

Решение 7

Конечно, некоторые читатели уже видят, что эту задачу можно легко решить с помощью комбинаторной формулы для определения числа сочетаний из 10 элементов, которые берутся по два за раз.



Впрочем, это решение, хотя оно эффективно, кратко и правильно, практически не требует математического мышления (если не считать применения формулы) и обходится без какого-либо подхода к решению задач. Несмотря на то, что такое решение имеет право на существование, только другие решения позволяют продемонстрировать различные стратегии, а именно с этой целью мы и привели данную задачу.

Мы предполагаем, что вы будете читать эту книгу, решать задачи и, таким образом, знакомиться со стратегиями. Это позволит вам составить собственный набор стратегий решения задач, который станет базовым в решении ваших задач. У тех, для кого решение задач является новым делом, мы надеемся пробудить интерес и подтолкнуть к дальнейшему изучению этого полезного аспекта математики. Те же, кто уже интересуется критическим мышлением и решением задач, найдут здесь новые, занятные и нестандартные задачи, способные захватить внимание. Приятного вам чтения!

Глава 1
Логическое рассуждение

Выделение целой главы такой стратегии, как логическое рассуждение, может показаться излишним. В самом деле, без логического мышления, хотя оно и используется для решения задач, немыслимо применение ни одной стратегии. Для многих людей решение задач является практически синонимом логического рассуждения, или логического мышления. Так зачем же тогда нужна эта глава, и зачем вообще выделять эту стратегию?

В повседневной жизни мы прибегаем к логическому рассуждению, когда спорим о чем-нибудь с кем-то. И это понятно — во время спора мы рассчитываем на то, что определенные доводы будут вызывать конкретную реакцию. На работе мы с помощью логической цепочки доводов добиваемся изменения того или иного производственного процесса. Мы логически выстраиваем цепочку утверждений в надежде на получение желаемого вывода. В суде, например, адвокаты используют логическое рассуждение, чтобы представить дело в нужном им свете. Если мы назначаем кому-то встречу через два дня, а сегодня суббота, то логика подсказывает нам, что встреча должна состояться в понедельник.

В математике некоторые задачи решаются без использования каких-либо других стратегий, включая и представленные в этой книге. Они требуют строгих рассуждений и формулирования утверждений, которые логически вытекают одно из другого. Возьмем, например, такую задачу.

Найдите все пары простых чисел, сумма которых равна 741.

Многие наверняка составят перечень всех простых чисел меньше 741 и будут подбирать к ним пару, дающую в сумме 741. Вместе с тем работу можно упростить с помощью логического рассуждения. Если сумма двух чисел является нечетным числом, то одно из слагаемых должно быть нечетным, а другое — четным. Как известно, существует только одно четное простое число — 2. Значит, другим числом должно быть 739 (а 739 — это простое число). Таким образом, мы нашли все пары, которые удовлетворяют условиям задачи.

Рассмотрим еще одну задачу, которая решается путем логического рассуждения.

Палиндромическим называют такое число, которое читается одинаково слева направо и справа налево. Примерами трехзначного и четырехзначного палиндромов являются 373 и 8668. Мария выписала все трехзначные палиндромы на листочки бумаги и положила их в большую коробку. Мигель выписал все четырехзначные палиндромы и положил листочки с числами в ту же коробку. Учитель тщательно перемешал листочки и попросил Лору взять один из них не глядя. Какова вероятность того, что она вытащит четырехзначный палиндром?

Один из способов решения — выписать все трехзначные и четырехзначные палиндромы, пересчитать их и определить искомую вероятность. Такой подход дает надежный результат, хотя и требует времени. Вместе с тем логическое рассуждение позволяет упростить работу. В качестве примера трехзначного палиндрома можно взять 373. Чтобы превратить его в четырехзначный палиндром, нужно всего лишь удвоить среднюю цифру — 3773. Повторяя это действие, мы можем превратить каждый трехзначный палиндром в четырехзначный. Таким образом, количество четырехзначных палиндромов равно количеству трехзначных, и вероятность выбора листочка с четырехзначным палиндромом составляет один из двух, или

Покажем еще на одном примере, насколько просто решаются задачи путем логического рассуждения.

На прилавке цветочного магазина стоят три коробки с декоративными бантиками для украшения подарочной упаковки. Марк решил пометить коробки ярлыками с надписями «Красные», «Белые» и «Разноцветные» (красно-белые). К сожалению, он наклеил эти ярлыки неправильно. Поскольку коробки стоят высоко, Марк не может заглядывать в них. Он знает, что коробки помечены неправильно, и хочет достать бантик из одной из них. Из какой коробки ему нужно достать бантик, чтобы пометить коробки правильно?

Давайте порассуждаем. Для начала заметьте, что все сказанное о коробке с ярлыком «Белые» в равной мере относится и коробке с ярлыком «Красные». Здесь существует своего рода симметрия. Поэтому, пусть Марк возьмет один бантик из коробки с ярлыком «Разноцветные». Если бантик окажется красным, то в этой коробке на самом деле находятся только красные бантики, поскольку они не разноцветные. Пометим ее как «Красные». Коробка с ярлыком «Белые» не может содержать чисто белые бантики, поэтому она должна получить ярлык «Разноцветные». Наконец, на коробку, ошибочно помеченную как «Красные», нужно наклеить ярлык «Белые».

Обратите внимание на то, что для решения каждой из рассмотренных задач необходимы всего лишь логическое рассуждение и размышление. Это ни в коей мере не означает, что логическое мышление не требуется при использовании других стратегий решения задач, однако задачи, представленные в этой главе, решаются почти исключительно путем логического рассуждения.

Задача 1.1

Макс начинает отсчитывать натуральные числа в порядке увеличения: 1, 2, 3, 4, …, а Сэм ведет отсчет с той же скоростью, но в обратном порядке от числа x: x, x — 1, x — 2, x — 3, x — 4, … Когда Макс доходит до 52, Сэм называет число 74. С какого числа (x) Сэм начал обратный отсчет?

Обычный подход

Столкнувшись с такой задачей, большинство людей обычно пытаются воспроизвести описанную ситуацию, т. е. выполнить одновременно процедуры отсчета, чтобы посмотреть, какой получится результат. Сложность здесь, однако, заключается в том, что начальное число для обратного отсчета неизвестно, поэтому, скорее всего, будут использоваться прямой отсчет и метод последовательного приближения. Это не только долго, но и очень трудно.

Образцовое решение

Подойдем к решению задачи логически. Макс отсчитал 52 числа, а значит и Сэм отсчитал такое же количество чисел. Можно представить 52-е число Сэма как x — 51. Как известно, это число равно 74. Таким образом, мы получаем уравнение x — 51 = 74, из которого следует, что x = 125.

Задача 1.2

У нас 100 кг свежих ягод, в которых 99 % массы приходится на воду. Через некоторое время содержание воды в ягодах уменьшается до 98 %. Сколько теперь весят ягоды?

Обычный подход

Чаще всего говорят, что после испарения 1 % воды вес ягод должен уменьшиться до 99 %, а значит ягоды весят 99 кг. Это неправильно!

Образцовое решение

Попробуем найти ответ путем логического рассуждения. Исходно в ягодах содержится 99 % воды, т. е. в них 99 кг воды и 1 кг сухого вещества, иначе говоря, масса сухих ягод составляет 1 %. Масса сухого вещества не меняется: в конце процесса сушки она так и останется равной 1 кг. Вместе с тем доля того, что не является водой, удваивается до 2 %.

Для того, чтобы нечто, имеющее фиксированное количество (1 кг сухого вещества в нашем случае), удвоило свою долю (с 1 % до 2 %), суммарное количество смеси должно уменьшиться в два раза. В начале у нас был 1 % сухого вещества, или а в конце — 2 %, или что сокращается до т. е. мы получаем 1 кг сухого вещества в 50 кг суммарной массы. Таким образом, в конце в ягодах остается 49 кг воды.

Задача 1.3

Во время школьного эксперимента Мигель многократно бросает обычный шестигранный игральный кубик. Он следит за каждой выпавшей цифрой и хочет остановиться, как только одна цифра выпадет три раза. Мигель останавливается после 12-го броска, и сумма выпавших цифр составляет 47. Какая цифра выпала третий раз? (Обычный шестигранный игральный кубик имеет цифры от 1 до 6.)

Обычный подход

Одно из решений — это взять игральный кубик и поэкспериментировать с ним. Получить точно 47 очков за 12 бросков довольно трудно, но даже если это и получится, то такое решение нельзя назвать изящным!

Образцовое решение

Давайте порассуждаем. За 11 бросков ни одна цифра не выпала три раза, иначе эксперимент закончился бы. Это означает, что пять цифр выпали дважды, а одна — лишь однократно. Обозначим эту цифру символом M. Если M выпадет в 12-м броске, то сумма будет равна 2 (1 + 2 + 3 + 4 + 5 + 6) = 42. Таким образом, сумма после 11 бросков составляет 42 — M. Если N — число, выпавшее в третий раз, то 42 — M + N = 47, а N — M = 5. Мы знаем, что N и M могут иметь значения только от 1 до 6. Единственные два числа из данного ряда, которые имеют разность 5, это 6 и 1. С учетом такого ограничения уравнение N — M = 5 имеет единственное решение, где M = 1, а N = 6. Таким образом, в третий раз выпала цифра 6.

Задача 1.4

Имеется треугольник, периметр которого численно равен его площади. Чему равен радиус вписанной в треугольник окружности?

Обычный подход

Обычно при решении этой задачи строят чертеж, как показано на рис. 1.1, и подбирают значения в попытке найти ответ. При таком подходе нужно быть готовым к разочарованиям.


Образцовое решение

Для решения этой задачи необходимо немного логики и следование поставленным условиям. Начнем с треугольника ABC, периметр которого равен p = AB + BC + CA. Обозначим символом O центр вписанной окружности с радиусом r. Площадь треугольника ABC равна сумме площадей треугольников AOB, BOC и COA с основаниями AB, BC и CA, соответственно, и высотой r. Это дает нам следующее уравнение:



Поскольку периметр треугольника численно равен его площади, мы получаем:

Задача 1.5

В США президентов выбирают каждые четыре года в годы, кратные 4. Некоторые из этих лет являются также квадратами целых чисел. Сколько президентских выборов между 1788 и 2016 годами пришлось на годы, которые являются квадратами простых чисел? В каких годах они проводились?

Обычный подход

Один из путей решения этой задачи — перебор всех четырехлетних периодов между 1788 и 2016 г. Поскольку 1788 делится на 4, то это будет первый год президентских выборов в рассматриваемом диапазоне. Таким образом, можно составить перечень этих лет (1788, 1792, 1796, …, 2012, 2016), а затем извлечь квадратный корень из каждого для определения тех лет, которые являются квадратами целых чисел. Калькулятор, конечно, облегчит задачу, но процесс решения все равно будет долгим и нудным!

Образцовое решение

Это отличный пример применения стратегии логического рассуждения. Прежде всего, кратным 4 может быть только четный год, поэтому можно отбросить все нечетные годы. Помимо этого, квадратные корни из этих лет должны лежать в интервале от 40 до 50, поскольку:

402 = 1600 (до заданного диапазона);

422 = 1764 (до заданного диапазона);

442 = 1936;

462 = 2116 (после заданного диапазона).

В пределах заданного диапазона находится только 1936 г. Таким образом, 1936 — это единственный год президентских выборов, который является квадратом целого числа.

Задача 1.6

Джимми подбрасывает одновременно две монетки. Он делает это до тех пор, пока хотя бы на одной монетке не выпадет орел (О). На этом игра заканчивается. Какова вероятность того, что в последнем подбрасывании орел выпадет на обеих монетках?

Обычный подход

Первая реакция — это взять две монетки и посмотреть, какими будут результаты после большого числа подбрасываний. Вместе с тем, как и в большинстве вероятностных экспериментов, пространство выборок чаще всего оказывается слишком маленьким, чтобы предсказать результат с приемлемой точностью.

Образцовое решение

Обратимся к стратегии логического рассуждения. При выполнении этого эксперимента все предыдущие подбрасывания монеток не имеют значения. Значение имеет только одно подбрасывание, в результате которого выпадает орел (О). Поэтому ограничимся анализом только этого последнего подбрасывания. Возможными являются четыре варианта:



В трех из этих четырех вариантов выпадает как минимум один орел. Орел не выпадает только в одном варианте — его можно отбросить. Единственный вариант с двумя орлами — это ОО. Таким образом, вероятность составляет

Задача 1.7

У одних пород свиней рождаются поросята с двумя завитками на хвостах, у других пород — с тремя завитками. Фермер поручает своим детям подсчитать, сколько свиней находится в свинарнике. Дети, одержимые математикой, сообщают ему, что количества свиней с двумя завитками и с тремя завитками выражаются простыми числами, а общее количество завитков на хвостах равно 40. Сколько свиней в свинарнике фермера?

Обычный подход

Если взять за x количество свиней с двумя завитками на хвостах, а за y — количество свиней с тремя завитками, то мы получаем уравнение 2x + 3y = 40. Это одно уравнение с двумя неизвестными. Числа здесь сравнительно невелики, поэтому можно попробовать найти ответ путем подстановки различных значений x и y. Вместе с тем, поскольку известно, что x и y простые числа, выбор ограничивается следующими величинами: 19, 17, 13, 11, 7, 5, 3 и 2. В любом случае процесс решения довольно длителен, скучен и громоздок.

Образцовое решение

Если взять за x количество свиней с двумя завитками на хвостах, а за y — количество свиней с тремя завитками, то 2x + 3y = 40, как мы уже говорили. Однако на этот раз пойдем дальше и проанализируем полученное уравнение, опираясь на логику. Поскольку и 40, и 2x — четные числа, четным числом должен быть и y, иначе сумма (40) не будет четной. Поскольку y — простое число, он должен быть равен 2 (это единственное четное простое число), а 3y должно равняться 6. Теперь решим уравнение для x:

2x + 6 = 40,

2x = 34,

x = 17.

У фермера в свинарнике 17 + 2, или 19 свиней.

Задача 1.8

Число называют «специальным», если оно делится на сумму составляющих его цифр. Какое из следующих чисел удовлетворяет этому условию?

11, 111, 1111, 11111, 111111, 1111111, 11111111, 111111111.

Обычный подход

Обычно мы подсчитываем сумму цифр в каждом числе и делим число на эту сумму. Например, 11 должно делиться на 1 + 1, или на 2. Но оно не делится на 2, поэтому 11 не является специальным числом. Если действовать таким образом, то нам придется решить восемь небольших задачек.

Образцовое решение

Хотя описанный выше подход в конечном итоге позволяет решить задачу, воспользуемся логическим рассуждением для поиска более изящного решения. Прежде всего, очевидно, что все приведенные числа являются нечетными, поскольку ни одно из них не оканчивается на 2, 4, 6, 8 и 0. Четное количество единиц даст нам четную сумму. Это позволяет отбросить числа с четной суммой единиц: 11, 1111, 111111 и 11111111. Помимо этого, число 11111 не делится на 5, поскольку оно не оканчивается на 0 или 5.

Если проверить число 1111111, то окажется, что оно не делится на 7. В результате у нас остаются всего два числа. Число 111 делится на 3, т. е. на сумму входящих в него цифр (3 x 37). Аналогичным образом число 111111111 делится на 9 (т. е. 9 x 12 345 679). Таким образом, 111 и 111111111 являются двумя «специальными» числами в приведенном числовом ряду.

Задача 1.9

Наименьшее число, которое делится на первые девять целых чисел, равно 2520. Какое наименьшее число будет делиться на первые 13 целых чисел?

Обычный подход

Проще всего найти все множители для первых 13 целых чисел и перемножить их. Это, правда, потребует много времени и утомительных вычислений. Не забывайте, что множители нельзя повторять (например, множитель 8 недопустим, поскольку 4 и 2 уже использовались). Так или иначе, данный метод позволяет в конечном итоге получить правильный ответ, если, конечно, все сделать тщательно и без ошибок.

Образцовое решение

Теперь попробуем порассуждать. Очевидно, что множители от 1 до 9 (первые девять целых чисел) уже использовались для получения произведения, равного 2520. Следовательно, нам нужно рассмотреть только целые числа 10, 11, 12 и 13, поскольку число 2520, задействующее предыдущие целые числа, уже известно. Множители 10 (5 x 2) и 12 (4 x 3) уже использовались. Однако 11 и 13 — это простые числа, которые делятся только сами на себя и на 1. Таким образом, умножив 2520 x 11 x 13, мы определяем, что наименьшее число, которое делится на первые 13 целых чисел, равно 360 360.

Задача 1.10

Ал, Барбара, Кэрол и Дэн сдают экзамен по математике. В целом они правильно ответили на 67 вопросов, и у каждого из них есть как минимум один правильный ответ. Ал дал больше всего правильных ответов. Барбара и Кэрол дали в сумме 43 правильных ответа. Сколько правильных ответов дал Дэн?

Обычный подход

Обычно делают предположение для каждого участника экзамена, проверяют, не нарушаются ли условия задачи, и смотрят, дают ли предположения в сумме 67. Такой подход может дать правильный ответ, однако все очень зависит от удачности предположений.

Образцовое решение

Применим нашу стратегию логического рассуждения. Поскольку Барбара и Кэрол вместе дали 43 правильных ответа, у одной из них таких ответов должно быть, как минимум, 22, а у другой — 21. Так как Ал оказался впереди всех, то с учетом предыдущих предположений в отношении Барбары и Кэрол у него должно быть, как минимум, 23 правильных ответа. Если допустить, что у Ала 23 правильных ответа, у Барбары — 22, а у Кэрол — 21, то в сумме у них будет 23 + 22 + 21 = 66 правильных ответов. Это означает, что Дэн правильно ответил только на один вопрос. Поскольку у всех есть как минимум один правильный ответ, результат 1 для Дэна правилен.

Задача 1.11

Лайза, которая едет на велосипеде по мосту, соединяющему точки A и B, и уже преодолела его длины, слышит, что сзади приближается поезд, движущийся со скоростью 60 км/ч. Она прикидывает расстояния и решает, что впритык сможет избежать столкновения, если поедет в любую сторону (к точке A или точке B) максимально быстро. Какова ее максимальная скорость?

Обычный подход

Поскольку длина моста неизвестна, зададим ее произвольно, выбрав какое-нибудь удобное (хотя, может быть, и нереалистичное) число, скажем, 8 км. Если Лайза поедет назад, к началу моста (точка A), со скоростью y км/ч, то она преодолеет 3 км за часа. За это время поезд пройдет x км от точки A. Данный отрезок времени можно представить, как Это дает нам уравнение: или xy = 180.

Если Лайза поедет к точке B, то аналогичным образом мы получим уравнение или xy + 8y = 300.

Объединив эти два уравнения, мы получим 8y = 300–180 = 120, а следовательно, y = 15.

Таким образом, максимальная скорость Лайзы равна 15 км/ч.

Образцовое решение

Стратегия логического рассуждения дает более изящное решение. Раз Лайза впритык успевает доехать до любого конца моста, будем считать, что она едет вперед к точке B. К тому моменту, когда поезд подойдет к точке A, она преодолеет еще пути, т. е. всего длины моста (или его длины). Теперь ей нужно проехать оставшуюся моста за то же самое время, которое требуется поезду, чтобы преодолеть полную длину моста. Таким образом, ее скорость равна скорости поезда, т. е. 15 км/ч.

Задача 1.12

Если S = 1! + 2! + 3! + 4! + 5! + … + 98! + 99! то какая цифра в числе S будет находиться в разряде единиц?

Напомним, что символ n! означает 1 x 2 x 3 x 4 x … x (n — 1) x n.

Обычный подход

Как правило, при решении такой задачи возникает желание определить значение каждого факториала, а затем сложить полученные значения и получить S. Помимо того, что это скучное занятие, оно еще чревато арифметическими ошибками.

Образцовое решение

Если проанализировать числовой ряд, составляющий S, и упростить его, то мы получим следующее:

S = 1! + 2! + 3! + 4! + 5! + … + 98! + 99!

S = 1 + 2 + 2 x 3 + 2 x 3 x 4 + 2 x 3 x 4 x 5 + … + 98! + 99!

S = 1 + 2 + 6 + 24 + 10k, где k — натуральное число.

Мы представили члены числового ряда, начиная с 5! как 10k, поскольку 5! предполагает наличие множителя 10. Любое число, кратное 5! будет кратно 10. Так как 6! кратно 5! а 7! кратно 6! то факториал любого n, превышающего 5, будет кратен 10. Таким образом, в разряде единиц будет находиться 0.

Глава 2
Распознавание закономерности

Одной из чудесных сторон математики является возможность выявления закономерностей в решаемых задачах. Известный математик Уолтер Сойер как-то заметил, что математику вполне можно представить, как процесс поиска закономерностей. Одно из самых распространенных применений математики — предсказание того, что происходит регулярным образом. Например, сколько пшеничных лепешек потребуется для трех человек? А для четырех? Для 10 человек? Для n человек?

Умение распознавать закономерности очень важно для решения задач. Выявив закономерность в результате анализа ряда конкретных примеров, вы можете обобщить ее и превратить в более широкое решение. Например, когда просят назвать следующие два числа в ряду 1, 2, 3, 6, 11, 20, 37, __, __, мы должны проанализировать ряд, чтобы понять, есть ли в числах какая-либо закономерность. В конце концов, если первые три члена это 1, 2, 3, то разве не 4 должно идти за ними? А вот и нет! Мы замечаем, что каждый член после третьего представляет собой сумму трех предшествующих чисел. (Это последовательность типа Фибоначчи.) Иначе говоря, 1 + 2 + 3 = 6, 2 + 3 + 6 = 11, 3 + 6 + 11 = 20 и т. д. Если продолжить ряд таким образом, то следующими двумя числами будут 11 + 20 + 37 = 68 и 20 + 37 + 68 = 125.

Даже маленькие дети пользуются закономерностями. Когда малыши начинают ходить в школу, они учатся считать. Закономерности помогают им вести счет единицами, потом двойками, пятерками и т. д. Если задать второкласснику вопрос, какое число будет следующим в ряду 3, 6, 9, 12, …, он спросит себя: «Сколько мне нужно прибавить к каждому числу, чтобы получить следующее?» Это практически естественное использование стратегии поиска закономерности.

Большинство из нас широко пользуются закономерностями в повседневной жизни. Некоторые из этих «закономерностей» требуют мнемонического подхода. Слово «мнемонический» происходит от древнегреческого слова mnemonikos, означавшего запоминающее устройство. Многие из нас знакомы с мнемоническим правилом запоминания порядка цветов в спектре «Каждый Охотник Желает Знать, Где Сидит Фазан» (красный, оранжевый, желтый, зеленый, голубой, синий, фиолетовый). Мы используем закономерности для запоминания кода замка шкафчика в раздевалке спортивного зала, телефонного номера и номерного знака автомобиля. В поисках дома с определенным номером мы почти интуитивно ожидаем увидеть нечетные номера на одной стороне улицы, а четные на другой — простая, но очень ценная закономерность.

Закономерности широко используются полицией. Если происходит серия преступлений, то следователь ищет стиль поведения преступников (modus operandi).

Врач обычно смотрит на характер поведения человека, чтобы определить его заболевание. Имея за плечами опыт лечения болезней, он распознает закономерные проявления недуга.

Эффективность стратегии распознавания закономерностей видна яснее всего на конкретных примерах, особенно когда не очевидно, что эту стратегию можно использовать для решения данной задачи. Допустим, вас просят найти цифру в разряде единиц у числа, представленного как 1323. Наиболее очевидный подход — взять калькулятор и возвести 13 в 23-ю степень. Однако это сложная задача, даже если есть калькулятор, способный воспроизвести количество разрядов такого огромного числа. Вместо этого можно проанализировать результаты возведения числа 13 в степень в порядке возрастания показателя и посмотреть, не образуют ли последние цифры какую-либо закономерность, помогающую дать ответ.



Похоже, при возведении числа 13 в степень последняя цифра образует ряд:

3, 9, 7, 1, 3, 9, 7, 1, …

Изменения происходят с периодом 4. Таким образом, число 1323 будет иметь ту же цифру в разряде единиц, что и 133, т. е. 7.

Фактически эта задача высвечивает интересный вопрос в отношении закономерностей. Можно ли утверждать, что при возведении всех чисел в степень цифра в разряде единиц изменяется циклически? Некоторые числа можно назвать сразу. Например, 5 в любой степени будет иметь в конце 5 (5, 25, 125, 625, …). Такое свойство чисел очень интересно и ценно для решения задач путем распознавания закономерности. Попробуйте определить закономерность изменения цифры в разряде единиц при возведении в степень других чисел.

Следует, однако, предостеречь читателей. Иногда случается, что закономерность вроде бы есть, но не вполне стабильная. Например, кажется, что любое нечетное число, начиная с 3, можно представить, как сумму 2 в той или иной степени и нечетного числа. При попытке проверить это практически оказывается, что данное «правило» выполняется вплоть до числа 125. Как ни странно, но оно не действительно для следующего нечетного числа 127. Таким образом, применять стратегию распознавания закономерности для решения задач следует с осторожностью. Впрочем, это всего лишь исключение, которое не должно удерживать вас от использования данного метода.

3 = 20 + 2

5 = 21 + 3

7 = 22 + 3

9 = 22 + 5

11 = 23 + 3

13 = 23 + 5

15 = 23 + 7

17 = 22 + 13

19 = 24 + 3

и так далее

51 = 25 + 19

и так далее

125 = 26 + 61

127 =?

129 = 25 + 97

131 = 27 + 3.

Перейдем теперь к задачам, которые наиболее эффективно решаются путем распознавания закономерности, особенно когда такая закономерность не очевидна.

Задача 2.1

Какая цифра находится в разряде единиц у числа, где — это показатели степени?

Обычный подход

К сожалению, находятся люди, которые полагают, что для определения значения этого числа нужно последовательно возвести основание в степень вплоть до последнего показателя. Такой подход не может быть успешным!

Образцовое решение

Попробуем выяснить, существует ли какая-то закономерность в числах по мере повышения показателя степени в соответствии с условиями задачи. По мере повышения показателя основания 2 цифры в разряде единиц изменяются в последовательности 2, 4, 8, 6.

21 = 2

22 = 4

23 = 8

24 = 16

25 = 32

26 = 64

27 = 128

28 = 256.

Результат на третьей ступени наших вычислений ниже кратен 4, а любой результат возведения 2 в степень, кратный 4, дает число, у которого в разряде единиц стоит 6.



Таким образом, у нашего числа в разряде единиц находится цифра 6.

Задача 2.2

В каждой приведенной ниже прямоугольной решетке содержится определенное количество точек. Сколько точек будет на рис. 49?


Обычный подход

Очевидный подход — это последовательное построение решеток вплоть до рис. 49, в котором можно подсчитать точки. Это займет много времени и потребует огромного терпения, не говоря уже о количестве бумаги. Вместе с тем наверняка должен существовать более практичный подход к решению этой задачи.

Образцовое решение

Попробуем организовать данные и поискать закономерность. Перенесем в таблицу то, что нам уже известно.



Ну вот и закономерность. Высота на 2 больше номера рисунка, а ширина на 1 больше номера рисунка. Для рис. n мы получаем:



Таким образом, на рис. 49 будет 51 x 50 = 2550 точек.

Задача 2.3

Круг можно разделить на семь частей с помощью трех прямых линий. Какое максимальное количество частей можно получить при делении круга с помощью семи прямых линий?

Обычный подход

Обычно при решении этой задачи берут круг и проводят через него семь линий так, чтобы любые три из них не пересекались, т. е. не имели общей точки. Если проделать такую операцию аккуратно, то она должна привести к правильному ответу. Вместе с тем определение максимально возможного количества частей может быть сложным.

Образцовое решение

При решении этой задачи интересно посмотреть, не проявится ли какая закономерность при увеличении количества линий, делящих круг на части, при условии, что никакие три из них не должны иметь общей точки. Понятно, что одна линия делит круг всего на две части. Две линии позволяют разделить круг на четыре части. В таблице ниже показано количество частей, на которые можно разделить круг с помощью заданного количества линий, ни одна тройка которых не имеет общей точки.



Закономерность, похоже, наблюдается в разнице, которая увеличивается каждый раз на единицу. Таким образом, протестировав следующий вариант, в котором пять линий предположительно дают 16 частей, мы можем, по всей видимости, составить на основе выявленной закономерности следующую таблицу.



Итак, с помощью семи линий можно разделить круг на 29 частей.

Задача 2.4

Нам дают карту с направлениями движения вдоль улиц, как показано на рис. 2.1.



Сколько существует маршрутов из точки A в точку L?

Обычный подход

Самый очевидный подход — просто подсчитать возможные маршруты. Иными словами, определять маршруты по одному за раз и суммировать результаты. Например, один маршрут — это A-B-C — D-E-F-G-H-I-J-K-L, другой — A-C-D-E-G-K-L и т. д. Вместе с тем, как вы видите, такой путь довольно громоздок, и к тому же при его использовании трудно избежать дублирования маршрутов. А вариантов здесь порядочно!

Образцовое решение

Воспользуемся стратегией поиска закономерности. Допустим, мы хотим попасть из точки A в точку B. Здесь имеется только один маршрут (A-B). В точку C можно добраться из точки A уже двумя путями (A-B-C и A-C). Из точки A в точку D существуют три маршрута, а именно (A-B-D, A-C-D, A-B-C-D). Если продолжить подсчет таким образом, то мы получим следующее количество маршрутов в каждую точку вплоть до точки F.



Они показаны на рис. 2.2.



Числовой ряд 1, 2, 3, 5, 8, 13 — это последовательность Фибоначчи, которую в западном мире впервые представил Леонардо Пизанский (известный так же, как Фибоначчи) в 1202 г. В начале такой последовательности стоят 1 и 1, а последующие числа получаются как сумма предыдущих двух. Если продолжить эту последовательность до точки L, то мы получим следующее:

1, 2, 3, 5, 8, 13, 21, 34, 55, 89, 144.

Таким образом, используя эту закономерность, мы находим, что из точки A до точки L можно добраться 144 маршрутами.

Задача 2.5

Джонни берет лист бумаги из записной книжки и разрывает его пополам, а затем кладет получившиеся части одну на другую и еще раз разрывает их пополам. Обрывки он опять складывает и рвет пополам. Если Джонни сможет повторить эту процедуру 20 раз, то какой толщины будет стопка обрывков? (Будем считать, что толщина листа бумаги 0,0254 мм.)

Обычный подход

Можно нарисовать таблицу и подсчитать результаты для каждого действия.



И так далее. В конечном итоге можно заполнить таблицу для всех 20 делений и найти ответ.

Образцовое решение

Воспользуемся стратегией поиска закономерности для решения этой задачи. После 1-го деления в стопке будет 2 слоя бумаги, после 2-го деления — 4 слоя, после 3 деления — 8 слоев. В экспоненциальной форме количество слоев можно представить, как 21, 22, 23, …, или 2n в общем виде. После 20 делений толщина стопки составит 0,0254 x 220, или около 26 645 мм, что составляет примерно 26,6 м. Вот почему в задаче говорится: «Если Джонни сможет повторить эту процедуру 20 раз».

Задача 2.6

Сколько квадратов всех размеров на стандартной шахматной доске размером 8 x 8 клеток?

Обычный подход

Первой реакцией будет ответ 8 x 8 = 64 квадрата, однако слова «всех размеров» говорят о том, что могут существовать и другие ответы. Математический подход предполагает подсчет количества квадратных областей всех размеров на шахматной доске с 64 клетками, т. е. 2 x 2, 3 x 3, 4 x 4 и т. д. Это неудобно и довольно трудно, поскольку перекрывается множество клеток. К тому же в процессе подсчета легко сбиться, так что такой метод скучен и проблематичен.

Образцовое решение

Попробуем применить стратегию поиска закономерности в сочетании с таблицей для организации данных. Если начать с доски размером 1 клетка на 1 клетку, то, очевидно, на ней будет только один квадрат, т. е. квадрат 1 x 1. На доске размером 2 клетки на 2 клетки мы увидим четыре квадрата 1 x 1 и один квадрат 2 x 2, т. е. всего 5 квадратов. Представим данные в таблице по мере увеличения размера нашей доски от 1 x 1 до 2 x 2, 3 x 3 и т. д.



В таблице явно просматривается закономерность заполнения клеток в каждой строке, поэтому мы быстро определяем, что на шахматной доске размером 8 x 8 клеток находятся 204 квадрата всех размеров.

В представленной выше таблице можно заметить не только одну закономерность. В ней, например, встречается множество квадратов целых чисел. А если взглянуть на колонку «Всего» и определить разность между следующими друг за другом членами, то мы получим интересную последовательность:

5–1 = 4

14–5 = 9

30–14 = 16

55–30 = 25

91–55 = 36

140–91 = 49

204–140 = 64.

Опять мы получаем квадраты целых чисел. Если теперь найти разность второго порядка, т. е. разность между квадратами, то мы получим последовательность нечетных чисел, начиная с 5:

9–4 = 5

16–9 = 7

25–16 = 9

36–25 = 11

49–36 = 13

64–49 = 15.

Закономерности не только очень полезны для решения задач, как мы видели выше, они также придают прелесть математике.

Задача 2.7

Таблица, представленная ниже, продолжается бесконечно. Какая буква будет находиться в середине 30-го ряда?


Обычный подход

Можно продолжить выписывать буквы в каждом ряду, пока не дойдем до 30-го ряда. Теперь можно определить, какая буква находится в середине. Такой метод громоздок, но он дает правильный ответ.

Образцовое решение

Это классический пример того, насколько эффективно поиск закономерности позволяет решать задачи. Для выявления закономерности построим еще четыре ряда букв.



Поскольку в последовательности 6 букв, ряды будут повторяться после каждых 6 букв. Более того, поскольку 30 кратно 6, буква в середине 30-го ряда будет той же самой, что и в середине 6 ряда, т. е. A. Стратегия распознавания закономерности делает решение задачи очень легким.

Задача 2.8

Найдите цифру в разряде единиц у каждого из следующих чисел:

a) 819;

b) 7197.

(Понятно, что это нужно сделать, не прибегая к помощи калькулятора или компьютера.)

Обычный подход

Некоторые пытаются решить эту задачу путем возведения 8 в степень с помощью калькулятора и очень быстро выясняют, что большинство калькуляторов не позволяет воспроизвести ответ такой величины. Количество разрядов на дисплее заканчивается раньше, чем на него будет выведено целевое значение.

Образцовое решение

Нам необходимо найти другой подход к решению этой задачи. Попробуем возводить 8 в последовательно увеличивающуюся степень и посмотрим, нет ли какой полезной закономерности в появлении последних цифр.



Обратите внимание на проявившуюся закономерность — цифра в разряде единиц повторяется при увеличении степени с шагом, равным четырем. По всей видимости, мы можем использовать эту закономерность при решении нашей задачи. Интересующая нас степень равна 19. При делении на 4 она дает остаток 3. Таким образом, последняя цифра числа 819 должна быть такой же, как и у 815, 811, 87 и 83, т. е. 2.

Для скептиков приведем фактическое значение 819 = 144 115 188 075 855 872.

Аналогичным образом проанализируем значения, получаемые при возведении 7 в последовательно увеличивающуюся степень, и попробуем отыскать закономерность.



В соответствии с этой закономерностью при делении показателя 197 на 4 мы получаем остаток, равный 1. Это означает, что последняя цифра числа 7197 должна быть такой же, как и у 71, т. е. 7. При наличии времени вы можете возвести 7 в степень 197 и проверить этот ответ. У вас должно получиться:


Задача 2.9

Чтобы составить квадрат 1 x 1, требуется 4 зубочистки, как показано на рис. 2.3.



Чтобы составить квадрат 2 x 2, требуется 12 зубочисток (рис. 2.4).



Сколько потребуется зубочисток, чтобы составить квадрат 7 x 7?

Обычный подход

Вы можете нарисовать квадрат 7 x 7 и просто подсчитать необходимое количество зубочисток. Такой подход вполне работоспособен, однако он громоздок и требует аккуратного построения чертежа.

Образцовое решение

Для начала попробуем построить несколько небольших квадратов и посмотрим, удастся ли нам выявить какую-либо закономерность. Нарисуем квадраты 3 x 3 и 4 x 4 (рис. 2.5 и 2.6).



Посмотрим теперь, что у нас получается.



Ну вот! При увеличении размера квадрата на 1 число необходимых зубочисток возрастает на 4. Продолжим таблицу:



Таблица показывает, что числа в третьей колонке последовательно возрастают на 4. Количество зубочисток можно определить в обратном порядке, зная результат из третьей колонки. Для создания квадрата 7 x 7 необходимо 112 зубочисток.

Глава 3
Действие от обратного

Само название этой стратегии приводит в замешательство большинство людей. Такой подход совершенно неестественен. Когда мы ходили в школу, нас учили решать математические задачи в прямом порядке. Как бы то ни было, многие задачи в реальной жизни решаются именно от обратного. В качестве простого примера предположим, что вам нужно забрать ребенка с тренировки точно в 17:00. Во сколько нужно выйти из дома? Допустим, чтобы добраться до стадиона, нужно 30 минут. По-хорошему, к этому следует добавить запас 5 минут. Значит, выйти нужно за 35 минут, или не позднее 16:25. Даже не задумываясь об этом, мы использовали действие от обратного! Конечно, это сильно упрощенный пример применения данной стратегии.

Чтобы лучше понять такой тип мышления, рассмотрим еще один пример. Допустим, произошла автомобильная авария. Полиции приходится действовать от обратного, чтобы восстановить сцену произошедшего. Кто в кого врезался? Какой автомобиль занесло? Как далеко тянутся следы шин на асфальте? У кого было преимущество в проезде? Это всего лишь один из множества примеров действия от обратного.

В случае применения подхода от обратного мы обычно начинаем с конца задачи, или с «ответа». От этой точки восстанавливаются необходимые действия. Так, если в задаче говорится «увеличилось на 2», мы «уменьшаем на 2», или вычитаем 2. Как-никак, если мы увеличили что-то на 2, то для возврата к предыдущему этапу нужно уменьшить это на 2. Аналогичным образом, если говорится об умножении на 3, то в случае действия от обратного, необходимо разделить результат на 3. Рассмотрим типичную задачу.

Средний результат Марии в 11 тестах равен 80. При определении итогового среднего результата учительница проявляет благосклонность и отбрасывает низший результат. В нашем случае она отбрасывает 30. Какой итоговый средний результат у Марии?

Будем двигаться от среднего результата Марии. Среднее (или среднее арифметическое) обычно определяется путем сложения всех результатов и деления суммы на количество результатов. Если средний результат 11 тестов равен 80, то сумма результатов 11 тестов должна составлять 11 x 80 = 880. (Обратите внимание на то, что мы умножаем на 11, т. е. выполняем обратное действие по отношению к первоначальному делению на 11.) Вычтем результат 30, который учительница отбросила, и уменьшим количество тестов на единицу. Таким образом, суммарный результат 10 тестов равен 850. Итоговый средний результат Марии равен:

850: 10 = 85.

Попробуем решить еще одну задачу с помощью действия от обратного.

Дэвид вернулся после четырех раундов игры в бейсбольные карточки. В его набор теперь входят 45 карточек. Когда я поинтересовался его успехами, он ответил, что потерял половину карточек в первом раунде. Во втором раунде он выиграл в 12 раз больше того, что было у него в тот момент. В третьем раунде выигрыш составил 9 карточек. Четвертый раунд закончился вничью, поэтому количество карточек у игроков не изменилось. Сколько карточек было у Дэвида перед началом игры?

Можно, конечно, составить ряд уравнений и попробовать решить задачу напрямую. Однако давайте посмотрим, сработает ли здесь наш подход от обратного. У нас есть конечный результат (45 карточек), а найти нужно начальное количество. Это своего рода «товарный знак» типичной задачи, эффективно решаемой с помощью вычисления от обратного. Итак, Дэвид закончил игру с 45 карточками. Четвертый раунд закончился вничью, поэтому в конце третьего раунда у него были все те же 45 карточек. В третьем раунде выигрыш составил 9 карточек, значит в конце второго раунда количество карточек было равно 36. Во втором раунде мальчик выиграл в 12 раз больше карточек, чем было, поэтому в конце первого раунда он должен был иметь 3 карточки. В первом раунде Дэвид проиграл половину своих карточек, таким образом, он начал игру с 6 карточками. Подход от обратного позволил легко решить эту задачу.

Задача 3.1

Сумма двух чисел равна 2. Произведение этих же двух чисел равно 5. Найдите сумму обратных величин этих двух чисел.

Обычный подход

Задача очевидно предполагает составление двух уравнений с двумя неизвестными:

x + y = 2;

xy = 5.

Эти два уравнения можно решить одновременно с использованием формулы корней квадратного уравнения: для ax2 + bx + c = 0. Однако этот метод дает значения для x и y в виде комплексных чисел, а именно 1 + 2i и 1–2i. В соответствии с условиями нашей задачи нам нужно найти сумму обратных величин этих двух квадратных корней.



Подчеркнем, что в таком методе нет ничего неправильного, это просто не самый изящный способ решения задачи.

Образцовое решение

Прежде чем браться за решение задачи, полезно отступить на шаг назад и посмотреть, что требуется. Заметим, что в данной задаче требуется определить не значения x и y, а сумму обратных величин этих двух чисел. Иначе говоря, нам нужно найти Используя подход от обратного, мы можем задаться вопросом, к чему это ведет. Сложение этих двух дробей может дать ответ. Таким образом, Фактически мы сразу получаем ответ, поскольку знаем, что сумма чисел равна 2, а их произведение — 5. Просто подставим эти значения в последнюю дробь и получим: Задача решена.

Задача 3.2

В распоряжении Лорен 11-литровый и 5-литровый сосуды. Как ей отмерить точно 7 литров воды?

Обычный подход

Большинство людей начинают строить догадки и «переливать воду» туда-сюда в попытке найти правильный ответ. Это своего рода «неинтеллектуальный» метод проб и ошибок.

Образцовое решение

Вместе с тем задачу можно решить более рационально при использовании подхода от обратного. В конечном итоге нам нужно получить 7 литров воды в 11-литровом сосуде, оставив свободным пространство объемом 4 литра. Откуда взялись эти 4 литра? (См. рис. 3.1.)




Чтобы получить 4 литра, мы должны оставить 1 литр воды в 5-литровом сосуде. Но как получить 1 литр в таком сосуде? Наполните 11-литровый сосуд водой и дважды отлейте воду в 5-литровый сосуд. В 11-литровом сосуде останется ровно 1 литр воды. Вылейте этот 1 литр в 5-литровый сосуд (рис. 3.2).

Теперь наполните 11-литровый сосуд и отлейте из него 4 литра воды в 5-литровый сосуд до его заполнения. В 11-литровом сосуде останутся требуемые 7 литров воды (рис. 3.3).



Учтите, что задачи подобного типа не всегда имеют решение. Иначе говоря, если вы хотите составить новую задачу такого вида, следует знать, что решение существует только в тех случаях, когда разница величин, кратных емкостям двух сосудов, может быть равной заданному объему. В нашем случае 2 x 11 - 3 x 5 = 7.

Задача 3.3

По определению, палиндром — это число, которое одинаково читается слева направо и справа налево. Так, числа 66, 595, 2332, 7007 являются палиндромами. Учитель Джека дал классу задание найти сумму первых 15 натуральных чисел. Джек взял калькулятор и сложил все числа от 1 до 15. Результат, к его удивлению, оказался палиндромом. Вместе с тем Джек пропустил одно число. Какое число он забыл включить?

Обычный подход

Как правило пытаются составить все возможные комбинации слагаемых, исключая по одному числу каждый раз, до тех пор, пока сумма 14 чисел не даст палиндром. Такой грубый метод вполне работоспособен, особенно когда вы используете калькулятор. Вместе с тем он требует времени, если вы действительно исключаете по одному числу за раз.

Образцовое решение

Попробуем подойти к решению задачи иначе и сначала определим, какую сумму должны дать первые 15 натуральных чисел. Хотя можно воспользоваться известной формулой для вычисления суммы членов арифметической прогрессии, а именно намного интереснее пойти путем, который предложил Карл Фридрих Гаусс, когда ему было 10 лет. Вместо того, чтобы складывать числа последовательно: 1 + 2 + 3 + … + 14 + 15, он к первому числу прибавил последнее, затем ко второму — предпоследнее и т. д. В результате у него получилось семь раз по 16 и 8 в середине, что в сумме составило 7 x 16 + 8 = 120.

Поскольку Джек упустил одно слагаемое и получил палиндром, результатом должно быть число 111. Вы можете возразить, почему именно этот палиндром, а не 101, например? Чтобы получить 101, упустив одно число, вы должны забыть 19, а это число лежит за пределами нашего интервала 1–15. Таким образом, Джек забыл число 9.

Задача 3.4

Мама испекла печенье на полдник для Берты. В первый день Берта съела половину всего испеченного печенья. На второй день она съела половину от того, что осталось. На третий день — одну четверть остатка, а на четвертый — одну треть. На пятый день она довольствовалась половиной того, что осталось, а на шестой день доела одно последнее печенье. Какое количество печенья испекла мама Берты?

Обычный подход

Первая реакция — написать ряд выражений, представляющих количество печенья, съеденного каждый день. Допустим, x — это начальное количество печенья.



Мама Берты испекла 16 печений.

Образцовое решение

Более эффективным является использование нашего подхода от обратного. Начнем с конца задачи и пойдем в обратном порядке:

В день 6 Берта съела одно последнее печенье, значит было 1 печенье;

В день 5 она съела 1/2, значит было 2 печенья;

В день 4 она съела 1/3, значит было 3 печенья;

В день 3 она съела 1/4, значит было 4 печенья;

В день 2 она съела 1/2, значит было 8 печений;

В день 1 она съела 1/2, значит было 16 печений.

Таким образом, вначале у Берты было 16 печений. Обратите внимание на то, что при вычислениях от обратного необходимо изменять используемые операции на «обратные». Вместо деления пополам мы должны удваивать, вместо сложения — вычитать и т. д. Это довольно легкий процесс.

Задача 3.5

Задача, которая ставит в тупик многих любителей математики, выглядит так: Марии 24 года. Она в два раза старше, чем была Анна, когда ей было столько же, сколько Анне сейчас. Сколько лет Анне?

Обычный подход

Для решения этой задачи недостаточно просто составить уравнение, которое даст ответ. Требуется нечто большее. Можно начать с создания таблицы, показанной на рис. 3.4.



Мы имеем 24 = 2a, следовательно a = 12. Кроме того, 24 - x = a + x = 12 + x, следовательно x = 6. Анне было 12, когда Марии было столько же (18), сколько Анне сейчас (18).

Образцовое решение

Подход от обратного может оказаться полезным для решения этой задачи. А раз так, то начнем со следующих рассуждений.

В представленной ситуации есть два временных периода:

1. Нынешнее время, когда Марии 24 года.

2. Прошлое время n лет назад.

Введем следующие обозначения:

M — возраст Марии (24), A — возраст Анны, n — разница между двумя временными периодами.

В первом временном периоде — Мария в два раза старше, чем была Анна:

2 (A - n) = M. (3.1)

Во втором временном периоде — когда Марии было столько же, сколько Анне сейчас:

M - n = A. (3.2)

Подставим уравнение 3.2 в уравнение 3.1:



Значение n = 6 при подстановке в уравнение 3.2 дает:

M - 6 = A -> A = 24 - 6 = 18.

Таким образом, возраст Анны составляет 18 лет.

Задача 3.6

От какой точки в выпуклом четырехугольнике сумма расстояний до каждой из вершин будет минимальной?

Обычный подход

Большинство без особых раздумий пытаются методом проб и ошибок найти точку, для которой сумма расстояний до вершин будет наименьшей. Вполне возможно, что кто-то выберет точку на пересечении диагоналей. Это правильный ответ, однако такой подход оставляет вопросы.

Образцовое решение

Наша стратегия поиска ответа от обратного оказывается более рациональной в данном случае. Возьмем четырехугольник ABCD с диагоналями, пересекающимися в точке E, и с точкой P, которая, на наш взгляд, может быть искомой, имеющей минимальную сумму расстояний до вершин. Соединим точку P пунктирными линиями с вершинами, как показано на рис. 3.5.



Рассмотрение треугольника APC показывает, что AP + PC > AC, поскольку сумма любых двух сторон треугольника всегда больше третьей стороны. Аналогичным образом, BP + PD > BD. В результате суммирования мы получаем, что AP + PC + BP + PD > AC + BD. Таким образом, отталкиваясь от предположения, что P может быть искомой точкой, мы находим, что выбор любой другой точки даст такой же результат. Единственной точкой, удовлетворяющей условиям задачи, является точка E на пересечении диагоналей.

Задача 3.7

Допустим, квадратные корни из уравнения x2 + 3x — 3 = 0 равны r и s. Чему равна сумма r2 + s2?

Обычный подход

Обычный подход заключается в решении уравнения для значений r и s. Используя формулу для определения корней квадратного уравнения вида ax2 + bx + c = 0, мы получаем:



Теперь нам нужно найти квадраты этих корней и их сумму:


Образцовое решение

Чтобы получить более изящное решение, нужно вспомнить зависимость из элементарной алгебры, в соответствии с которой сумма корней квадратного уравнения ax2 + bx + c = 0 составляет а произведение корней Из приведенного в условиях задачи уравнения мы находим, что сумма корней r + s = –3, а произведение rs = –3. При подходе от обратного, т. е. при определении суммы квадратов корней вместо прямых вычислений, как мы делали выше, для определения корней нам нужно искать эту сумму, поскольку (r + s)2 = r2 + s2 + 2rs. Перепишем это уравнение следующим образом r2 + s2 = (r + s)2 — 2rs.

Таким образом, значение r2 + s2 = (–3)2 — 2 (–3) = 9 + 6 = 15.

Задача 3.8

Макс, Сэм и Джек играют в необычную карточную игру. В этой игре проигравший отдает другим игрокам столько денег, сколько у них есть. Макс проигрывает в первой партии и отдает Сэму и Джеку столько денег, сколько есть у каждого из них. Сэм проигрывает во второй партии и отдает Максу и Джеку столько денег, сколько есть у каждого из них. Джек проигрывает в третьей партии и отдает Максу и Сэму столько денег, сколько есть у каждого из них. На этом они решают закончить игру, и у каждого остается ровно $8,00. Сколько денег у каждого из игроков было перед началом игры?

Обычный подход

Задача предполагает составление ряда уравнений, представляющих каждую партию. Обозначим начальную сумму денег у каждого игрока следующим образом: Макс — x, Сэм — y, Джек — z.



В последней партии, как мы знаем, каждое из значений равно 8. Это дает следующие три уравнения с тремя неизвестными:



В результате решения системы из трех уравнений мы получаем:

x = 13, y = 7, z = 4.

Образцовое решение


Обратите внимание, что в задаче дается конечная ситуация и спрашивается, какой была начальная ситуация. Это может указывать на эффективность подхода от обратного при решении. Также заметьте, что в соответствии с описанием ситуации в игре постоянно находится одно и то же количество денег (а именно $24). Подход от обратного дает изящное решение.

Макс начинает с $13, Сэм — с $7, а Джек — с $4. Ответ получился таким же, как и при обычном подходе, однако решение было более изящным.

Задача 3.9

Ал и Стив делят пятнистых саламандр для участия в выставке. Ал отбирает для своей экспозиции саламандр с двумя пятнами, а Стив — с семью пятнами. У Ала на пять саламандр больше, чем у Стива. Всего на их саламандрах 100 пятен. Сколько саламандр на двух экспозициях?

Обычный подход

Характер этой задачи создает сложности для использования алгебры. Обычно количество саламандр у Ала обозначают как x, а количество саламандр у Стива — как y. Это позволяет составить следующие уравнения:

x — y = 5;

2x + 7y = 100.

Для решения этих двух уравнений умножим первое из них на 2:

2x — 2y = 10;

2x + 7y = 100.

Вычитание одного уравнения из другого дает следующий результат:

9y = 90, или y = 10.

Теперь подставим значение y в первое уравнение и получим x = 15. Таким образом, у Ала и Стива вместе 15 + 10 = 25 саламандр. Это решение абсолютно правильное, но не самое изящное.

Образцовое решение

Посмотрим, можно ли упростить решение, использовав подход от обратного. Нас не спрашивают, сколько саламандр у каждого мальчика, мы должны определить сумму их саламандр. Поэтому можно начать с тех же двух уравнений. Иначе говоря, нам нужно найти x + y, а не значение каждой неизвестной. Составим те же два уравнения исходя из условий задачи.

x — y = 5;

2x + 7y = 100.

В этот раз, однако, будем искать способ определения суммы двух неизвестных.

Для этого умножим первое уравнение на 5, а второе на 2:

5x — 5y = 25;

4x + 14y = 200.

Теперь сложим эти два уравнения и получим 9x + 9y = 225 и x + y = 25. Такой метод необычен, но он демонстрирует более тонкий подход к решению задач, в которых требуется найти нечто иное, чем ожидают большинство людей.

Задача 3.10

Имея два следующих уравнения, найдите сумму x + y:

6x + 7y = 2007;

7x + 6y = 7002.

Обычный подход

Традиционный подход заключается в решении двух уравнений с двумя неизвестными.

6x + 7y = 2007;

7x + 6y = 7002.

Умножим первое уравнение на 7, а второе на 6 и получим:

42x + 49y = 14 049;

42x + 36y = 42 012.

Вычтем одно уравнение из другого:

13y = -27 963.

Таким образом, y = -2151.

Подставив это значение y в первое уравнение, мы получаем:

6x - 15 057 = 2007;

6x = 17 064;

x = 2844.

Таким образом, искомая сумма равна x + y = 2844 - 2151 = 693.

Образцовое решение

Подойдем к решению этой задачи от обратного. Два уравнения, приведенные в условиях задачи, обладают определенной симметрией. Попробуем выяснить, не поможет ли эта симметрия найти более изящное решение. Глядя на вопрос задачи, можно заметить, что нам нужно найти не индивидуальные значения x и y, как обычно, а только их сумму. Поэтому давайте посмотрим, позволяет ли упомянутая выше симметрия найти сумму сразу без предварительного определения значений x и y. Если сложить два уравнения, мы получим:



Разделив обе части уравнения на 13, мы получаем x + y = 693, а это и есть искомый ответ.

Глава 4
Принятие другой точки зрения

Среди множества стратегий решения математических задач есть такая, которая позволяет выйти из положения, когда вы «упираетесь в стену». Это подход к задаче с другой точки зрения. Ниже приведен пример такой стратегии, который является классическим в силу простоты и кардинальности изменения метода решения. В этом примере обычный подход дает правильный ответ, однако он громоздок и нередко приводит к арифметическим ошибкам. Рассмотрим следующую задачу.

В школе 25 классов, каждый из которых выставляет баскетбольную команду для участия в общешкольном турнире. По условиям турнира команда, проигравшая в одной встрече, выбывает из соревнования. В школе всего один спортивный зал, и директор хочет знать, сколько встреч придется провести в нем, чтобы определить победителя.

Типичное решение этой задачи заключается в моделировании реального турнира, в котором 12 случайно выбранных команд встречаются с другой группой из 12 команд, а одна команда освобождается от соревнований. Победители затем встречаются друг с другом, как показано ниже.

Любые 12 команд играют против других 12 команд, в результате чего определяются 12 победителей.

Во втором круге 6 победителей встречаются с 6 другими победителями, в результате чего определяются 6 победителей.

В третьем круге 3 победителя встречаются с 3 другими победителями, в результате чего определяются 3 победителя.

3 победителя + 1 команда (освобожденная от соревнований) = 4 команды.

В четвертом круге 2 оставшиеся команды встречаются с 2 оставшимися командами, в результате чего определяются 2 победителя турнира.

В пятом круге 1 команда играет против 1 команды за звание чемпиона.

Теперь подсчитаем количество проведенных игр.



Суммарное количество проведенных игр равно:

12 + 6 + 3 + 2 + 1 =24.

Такой метод решения кажется вполне разумным и определенно правильным.

Если подойти к этой задаче с другой точки зрения и определять проигравших, а не победителей, то решение будет значительно проще. В этом случае мы задаемся вопросом, сколько должно быть проигравших, чтобы определить одного чемпиона? Понятно, что проигравших должно быть 24. Чтобы появились 24 проигравших, нужно провести 24 игры. Ответ найден. Взгляд на задачу с другой точки зрения — интересный подход, который может оказаться полезным в различных ситуациях.

Для получения еще одной альтернативной точки зрения на задачу представьте, что в составе наших 25 команд одна является профессиональной баскетбольной командой, которая гарантированно побеждает в турнире. Каждая из оставшихся 24 команд при встрече с профессиональной командой неизбежно проигрывает. И опять мы видим, что для определения чемпиона нужно провести 24 игры. Это должно показать вам действенность данного метода решения задач. Посмотрим теперь, какие задачи можно эффективно решать с помощью принятия другой точки зрения.

Задача 4.1

На контуре круга O выбрана точка P (рис. 4.1). Из этой точки к взаимно перпендикулярным диаметрам проведены перпендикуляры PA и PB. Если AB = 12, то чему равна площадь круга, выраженная через ??

Обычный подход

Большинство пытается решить задачу с помощью теоремы Пифагора, поскольку треугольники PAB и OAB являются прямоугольными. Такой подход, однако, заводит в тупик из-за того, что для применения теоремы Пифагора недостаточно информации.


Образцовое решение

Эту задачу можно решить несколькими способами. Один из них — рассмотрение экстремумов. Предположим, что точка P на контуре круга совпадает с точкой Q. В этом случае отрезок AB должен совпадать с отрезком QO, который представляет собой радиус круга. Таким образом, площадь круга равна 144?.

Задачу также можно решить, посмотрев на нее с другой точки зрения. Четырехугольник с тремя прямыми углами является прямоугольником. Отрезок AB — диагональ прямоугольника. PO также диагональ прямоугольника. Поскольку диагонали прямоугольника равны, радиус круга PO = 12, а площадь круга равна 144?.

Задача 4.2

Стандартную колоду из 52 игральных карт делят случайным образом на две стопки по 26 карт в каждой. Как количество красных карт в одной стопке соотносится с количеством черных карт в другой?

Обычный подход

При решении этой задачи, как правило, количество черных и красных карт в каждой стопке представляют следующим образом:

B1 = количество черных карт в стопке 1;

B2 = количество черных карт в стопке 2;

R1 = количество красных карт в стопке 1;

R2 = количество красных карт в стопке 2.

Поскольку общее количество черных карт равно 26, можно записать, что B1 + B2 = 26, а поскольку общее количество карт в стопке 2 равно 26, мы получаем R2 + B2 = 26.

Вычитание второго уравнения из первого, дает: B1 — R2 = 0. Таким образом, B1 = R2, т. е. количество красных карт в одной стопке равно количеству черных карт в другой. Хотя такое решение дает ответ, назвать его изящным нельзя. Наша цель в этой главе — найти такое решение, которое демонстрирует красоту и силу математики.

Образцовое решение

В качестве альтернативы возьмем все красные карты из стопки 1 и обменяем их на черные карты из стопки 2. Теперь все черные карты находятся в одной стопке, а красные — в другой. Таким образом, количество красных карт в одной стопке и количество черных карт в другой стопке должны быть равными. Задачу позволяет решить простая логика — нужно лишь взглянуть на эту задачу с другой точки зрения.

Задача 4.3

Лоэнгрину дали четыре отрезка цепи (рис. 4.2), в каждом из которых три звена. Покажите, как соединить эти четыре отрезка в замкнутую цепь, разомкнув и сомкнув не более трех звеньев.


Обычный подход

Обычно сначала пытаются разомкнуть последнее звено одного отрезка, присоединить его к другому отрезку и получить цепь из 6 звеньев. Затем размыкают звено третьего отрезка и присоединяют его к 6-звенной цепи, получая 9-звенную цепь. Присоединив последний отрезок к 9-звенной цепи, получают 12-звенную цепь, которая, однако, не является замкнутой. Таким образом, традиционный подход обычно завершается неудачей. Некоторые пробуют другие комбинации размыкания/замыкания звеньев каждого отрезка цепи, однако такой подход не приносит желаемого результата.

Образцовое решение

Эта задача хорошо решается с помощью стратегии принятия другой точки зрения. Можно даже сказать, что такой подход просто неоценим в данном случае. Вместо того, чтобы пытаться разомкнуть и замкнуть одно звено на каждом отрезке цепи, другая точка зрения предполагает размыкание всех звеньев одного отрезка цепи и использование этих звеньев для соединения трех оставшихся отрезков цепи и получения требуемой замкнутой цепи. Это быстро приводит к правильному решению.

Задача 4.4

Какие натуральные числа менее 100 дают остаток 3 при делении на 7 и остаток 4 при делении на 5?

Обычный подход

Рассмотрим ряд натуральных чисел менее 100, которые дают остаток 3 при делении на 7: {3, 10, 17, 24, 31, 38, 45, 52, 59, 66, 73, 80, 87, 49}. Теперь рассмотрим ряд натуральных чисел менее 100, которые дают остаток 4 при делении на 5: {4, 9, 14, 19, 24, 29, 34, 39, 44, 49, 54, 59, 64, 69, 74, 79, 84, 89, 94, 99}.

Сравнив эти два ряда, находим три совпадающих числа: 24, 59 и 94.

Образцовое решение

Попробуем взглянуть на эту задачу с другой точки зрения. Искомые числа должны иметь форму 7n + 3, а также форму 5k + 4, где n и k — целочисленные неизвестные. Объединим эти величины так, чтобы искать числа в форме 35r + p, где r и p — целочисленные неизвестные. Первый ряд чисел, имеющих форму 7n + 3, также можно представить, как 35r + 3, 35r + 10, 35r + 17, 35r + 24 и 35r + 31. Только одно из этих чисел имеет также форму 5k + 4, а именно 35r + 24. Для того, чтобы узнать, какие числа менее 100, удовлетворяют этому соотношению, зададим r = 0, 1 и 2 и получим три искомых числа: 24, 59 и 94.

Задача 4.5

Какое из следующих двух выражений больше, ?5 + ?8 или ?4 + ?10

Обычный подход

Учитывая нынешнее распространение калькуляторов, не удивительно, что люди обычно извлекают квадратный корень из каждого числа, затем определяют их суммы и получают требуемый ответ. Хотя такой подход довольно эффективен, его, конечно, не назовешь изящным.

Образцовое решение

Взглянем на задачу с другой точки зрения, а именно возведем в квадрат каждую из этих сумм и посмотрим, каким будет результат.



Упростив, таким образом, условия, мы видим, что сумма является большей из двух величин.

Задача 4.6

Чему равны все положительные целочисленные значения переменной n, для которой дробь также является целым числом?

Обычный подход

Первой реакцией на эту задачу является попытка подставить разные значения n и посмотреть, какой результат будет целым числом. Например, если принять n = 4, мы получим т. е. целое число. Хотя такой подход и позволяет выявить некоторые значения n, очень трудно сказать, все ли значения найдены. В результате обычно получается неполный ответ.

Образцовое решение

Воспользуемся стратегией принятия другой точки зрения. Для начала выполним деление:



Чтобы эта величина была целым числом, n — 3 должно быть пропорционально 36. Делителями для числа 36 являются 1, 2, 3, 4, 6, 9, 12, 18 и 36. Таким образом:



Значения n, при которых является целым числом, равны 4, 5, 6, 7, 9, 12, 15, 21 и 39.

Задача 4.7

Каждый из 10 придворных ювелиров дает королевскому советнику г-ну Саксу стопку золотых монет. В каждой стопке находится 10 монет. Полноценные монеты весят 1 унцию. Однако в одной из стопок находятся «неполновесные» монеты, каждая из которых весит на 0,1 унции меньше. Г-н Сакс хочет выявить ювелира-жулика и стопку неполновесных монет с помощью всего лишь одного взвешивания. Как это сделать?

Обычный подход

Традиционная процедура начинается со случайного выбора стопки и ее взвешивания. Такой метод проб и ошибок дает искомый результат всего в 1 случае из 10. Учитывая это, можно попытаться решить задачу путем логического рассуждения. Прежде всего, если все монеты полновесные, их общий вес должен составлять 10 x 10 = 100 унций. Каждая из 10 неполновесных монет имеет меньший вес, поэтому недостача должна составить 10 x 0,1 = 1 унцию. Однако подход с точки зрения общей недостачи ничего не дает, поскольку она может оказаться в любой из стопок — в первой, второй, третьей и т. д.

Образцовое решение

Попробуем решить задачу, организовав данные иначе. Нам необходимо найти такой метод обнаружения недостачи, позволяющий идентифицировать стопку, из которой взяты неполновесные монеты. Присвоим стопкам номера № 1, № 2, № 3, № 4, …, № 9, № 10. Затем возьмем одну монету из стопки № 1, две монеты из стопки № 2, три монеты из стопки № 3, четыре монеты из стопки № 4 и т. д. Всего у нас получилось 1 + 2 + 3 + 4 + … + 8 + 9 + 10 = 55 монет. Если все монеты полновесные, то их общий вес должен составить 55 унций. Если обнаружится недостача 0,5 унции, значит в навеске присутствуют 5 неполновесных монет из стопки № 5. Если обнаружится недостача 0,7 унции, значит в навеске присутствуют 7 неполновесных монет из стопки № 7 и т. д. Таким образом, г-н Сакс может легко определить стопку неполновесных монет и ювелира, который принес эти монеты.

Задача 4.8

Ресторан быстрого питания продает куриные наггетсы в коробках по 7 штук и по 3 штуки. Какое наибольшее количество наггетсов нельзя купить?

Обычный подход

Мы просто пытаемся найти ответ путем перебора сочетаний 7 и 3 до тех пор, пока не дойдем до точки, начиная с которой можно купить любое количество наггетсов.



По всей видимости, наибольшее количество наггетсов, которое нельзя купить, равно 11. После этого все, что нужно, это добавлять 3 или 7.

Образцовое решение

Здесь мы обратимся к идее, привносящей в решение определенное изящество, и предоставим читателю возможность самому разобраться, почему это так, а не иначе. Существует теорема, известная под названием «теорема макнаггетсов». В соответствии с ней, если McDonald's продает макнаггетсы в коробках по a или b штук, где a и b не имеют общих множителей, то наибольшее количество макнаггетсов, которое нельзя купить, равно ab — (a + b). Например, если они продаются в коробках по 8 и 5 штук, то наибольшее количество макнаггетсов, которое нельзя купить, составляет 8 x 5 — (8 + 5) = 40–13 = 27.

В нашей задаче, наибольшее количество наггетсов, которое нельзя купить, равно 3 x 7 — (3 + 7), или 21–10 = 11.

Задача 4.9

Упростите каждое из следующих выражений:


Обычный подход

Хотя есть соблазн взять калькулятор и вычислить значение этих выражений, нередко наши надежды не оправдываются, и мы получаем на табло лишь сообщение error.

Образцовое решение

Подойдем к решению этой задачи с другой точки зрения. Учитывая, что число 3 возводить в степень довольно просто, решим задачу следующим образом:



Второе выражение можно упростить, разбив числа на простые множители следующим образом:


Задача 4.10

И у Вольфганга, и у Людвига есть целое число евро, причем каждое из них меньше 100. Когда они посчитали свои деньги, оказалось, что три четверти суммы Вольфганга равны двум третям суммы Людвига. Какое максимальное число евро может быть у каждого из них?

Обычный подход

Первая реакция — применить алгебраический подход. Мы можем составить одно уравнение с двумя неизвестными. Пусть W представляет количество евро у Вольфганга, а L — количество евро у Людвига. Наше уравнение имеет следующий вид:



Умножим обе части уравнения на 12 и получим: 9W = 8L. Решение уравнения для W дает следующий результат:



Поскольку у каждого из мальчиков по целому числу евро, сумма Людвига должна быть кратной 9, т. е. 9, 18, 27, 36, …, 99. Теперь можно подставить каждое из этих чисел в уравнение и определить количество евро у Людвига. Наибольшее количество евро, которое может иметь Людвиг, составляет 11 x 9, или 99 евро (менее, чем 100). Мы знаем, что суммы Людвига (66 евро) равны суммы Вольфганга. Таким образом, сумма Вольфганга составляет или 88 евро, а сумма Людвига — 99 евро.

Образцовое решение

Воспользуемся арифметическим подходом и взглянем на задачу с другой точки зрения. Поскольку суммы Вольфганга равны суммы Людвига, найдем эквивалентные дроби с одинаковым числителем:



Если у Вольфганга 8 евро, а у Людвига 9 евро, то части их сумм становятся одинаковыми и равными 6 евро. Поэтому ответ должен быть равен произведению одного и того же множителя на 8 и 9. Таким образом, наибольшая сумма, которую может иметь Людвиг, составляет 11 x 9, или 99 евро, а наибольшая сумма Вольфганга — 11 x 8, или 88 евро.

Ответ можно проверить, определив величину от 88 евро (66) и от 99 евро (66).

Задача 4.11

На рис. 4.3 ширина прямоугольника AEFK равна AK = 8, а длина AE разделена на четыре части AB = 1, BC = 6, CD = 4 и DE = 2. Чему равна площадь четырех закрашенных треугольников?


Обычный подход

Очевидный подход — найти площадь каждого из четырех треугольников и сложить их. Во всех четырех случаях высота треугольника равна ширине прямоугольника AK = 8. Таким образом, площади четырех треугольников составляет:



Сумма этих площадей равна 4 + 24 + 16 + 8 = 52.

Образцовое решение

Воспользуемся нашей стратегией принятия другой точки зрения на решение задачи. Треугольники имеют одну и ту же высоту, а именно 8. Сумма оснований четырех треугольников равна длине прямоугольника, т. е. 13. Таким образом, площадь четырех закрашенных треугольников равна половине площади прямоугольника, или

Задача 4.12

Определите, сколько чисел можно составить из цифр от 1 до 9 при условии, что цифры в этих числах должны располагаться в порядке возрастания.

Обычный подход

Большинство людей, скорее всего, воспользуются методом проб и ошибок и попытаются выяснить, нет ли здесь какой закономерности, и будут добавлять в список одно число за другим, т. е. сначала однозначные числа, затем двухзначные, трехзначные и т. д. Если выполнить эту работу тщательно, то можно получить правильный ответ, однако такой подход трудоемок.

Образцовое решение

Рассмотрим сначала набор целых чисел, имеющихся в нашем распоряжении {1, 2, 3, 4, 5, 6, 7, 8, 9}. Каждое подмножество этих цифр, за исключением пустого, должно давать одно из искомых чисел. Например, подмножество {3, 5, 7, 9} дает число 3579. Вопрос в том, сколько таких подмножеств можно выделить в нашем ряду из девяти цифр. Их количество равно 29 = 512. Вместе с тем сюда вошло пустое подмножество, которое необходимо вычесть. Таким образом, мы получаем 29 — 1 = 511 подмножеств из 9 цифр, каждое из которых дает число, где в соответствии с условием задачи, цифры могут располагаться в порядке возрастания.

Задача 4.13

На рис. 4.4 показан равнобедренный треугольник с бесконечным рядом окружностей, каждая из которых касается двух равных сторон треугольника и соседних окружностей, а нижняя окружность касается основания треугольника. Стороны равнобедренного треугольника равны 13, 13 и 10. Чему равна сумма длин этих окружностей?


Обычный подход

Занудный по определению подход предполагает вычисление длины каждой окружности с последующим определением суммы их длин. Подсчеты в этом случае очень трудоемки, но при тщательном выполнении они могут дать правильный ответ.

Образцовое решение

Воспользуемся стратегией рассмотрения задачи с другой точки зрения. С помощью теоремы Пифагора находим, что высота равнобедренного треугольника равна 12. Заметим, что сумма диаметров бесконечного числа окружностей равна высоте равнобедренного треугольника. Таким образом, сумма длин окружностей равна сумме диаметров, умноженной на ?, т. е. 12?.

Задача 4.14

Чему равен наименьший неотрицательный остаток при делении 227 на 123?

Обычный подход

Как правило, при решении этой задачи люди тратят кучу времени на определение значения числа 227, а потом делят результат на 123.

Образцовое решение

Мы подойдем к решению задачи с другой точки зрения. Вместо развертывания 227 в число без степени разложим его на числа в степени:

227 = (27) (117) = (27) (112) (112) (112) (11) = (123 + 5) (123–2) (123–2) (123–2) (11).

Теперь вспомним, что произведение двух двучленов вида 123 + s и 123 + t можно представить как 123k + st:

(123 + s) (123 + t) = 1232 + 123s + 123t + st = 123 (123 + s + t) + st = 123k + st.

Таким образом, мы получаем:

123n — 440 = 123n - 492 + 52 = 123 (n - 4) + 52.

При делении числа 227 на 123 остаток равен 52.

Задача 4.15

Во время футбольного матча команды получают 2 очка за сейфти, 3 очка за гол в ворота и 7 очков за тачдаун. Если отбросить 2 очка за сейфти, то команды смогут получать лишь по 3 и по 7 очков. Каково максимальное значение счета, которое нельзя получить в этом матче?

Обычный подход

Очевидный подход — выписывать все возможные значения счета до тех пор, пока не обнаружится максимальное значение, которое невозможно получить. Такой метод, однако, не дает уверенности в том, что не существует более высокое значение.

Образцовое решение

В этом случае можно воспользоваться стратегией принятия другой точки зрения. Вместо поисков значений счета, которые нельзя получить, определим значения, которые можно получить. Счет, который можно набрать, зарабатывая очки на голах в ворота, составляет 3, 6, 9, 12, 15, … Счет, который можно заработать на очках за тачдаун, составляет 7, 14, 21, 28, … Другие значения получаются в результате прибавления очков за гол в ворота или за тачдаун к предыдущему счету. Таким образом, значения, которые нельзя получить, составляют 2, 4, 5, 8, 11. Любой счет, начиная с 12, является доступным, как видно из следующего:



Таким образом, наивысший счет, который нельзя получить, равен 11.

Интересно отметить, что эта ситуация описывается чисто математически.

Наивысший счет, который нельзя получить при использовании двух простых чисел (a и b), равен произведению этих чисел за вычетом их суммы. В нашем случае это (7 x 3) - (7 + 3) = 11.

Задача 4.16

Число 6! (читается как «шесть факториал») равно произведению 6 x 5 x 4 x 3 x 2 x 1 = 720. Найдите значение

Обычный подход

Обычно так и подмывает выписать все факториалы, взять калькулятор или компьютер и вычислить фактические результаты. Это, конечно, позволит получить ответ, но потребует массы арифметических расчетов.

Образцовое решение

Давайте применим стратегию принятия другой точки зрения. Каждый из факториалов можно представить как число, кратное 98! Например, 100! можно записать как 100 x 99 x 98! а 99! — как 99 x 98! В результате мы получаем:



что является ответом этой задачи.

Задача 4.17

При делении 450 на нечетное число частное представляет собой простое число без остатка. Чему равно нечетное число?

Обычный подход

Обычно 450 последовательно делят на нечетные числа (1, 3, 5, …) до тех пор, пока не найдут частное в виде простого числа. Это в конечном итоге дает результат, но может потребовать довольно много времени.

Образцовое решение

Воспользуемся нашей стратегией и посмотрим на задачу с другой точки зрения. Число 450 можно записать как 2 x 32 x 52. Поскольку 32 и 52 это нечетные числа, а 450 — четное число, то единственным возможным четным простым множителем для 450 является 2. Таким образом, нечетное число равно 32 x 52 = 225.

Задача 4.18

Число 1 000 000 имеет множество пар целочисленных множителей, т. е. двух чисел, произведение которых равно 1 000 000. Однако существует только одна пара множителей, которые не содержат нулей. Чему равны эти множители?

Обычный подход

Традиционный подход — это перебор пар чисел, произведение которых равно 1 000 000, в поисках пары, не содержащей нулей. Можно начать с 1 x 1 000 000, 2 x 500 000 и т. д. Это наверняка потребует много времени — ведь у числа 1 000 000 масса пар множителей.

Образцовое решение

Проанализируем число 1 000 000 с другой точки зрения. Это число можно представить как 106. Степень, в свою очередь, можно разложить следующим образом: (2 x 5)6 = 26 x 56. Это дает нам два множителя, которые не содержат нулей: 26 = 64 и 56 = 15 625. Обратите внимание на то, что все остальные пары множителей должны содержать как минимум один ноль, поскольку при перемножении чисел 2 и 5 получается 10, а это дает число, оканчивающееся на ноль.

Глава 5
Анализ экстремальных ситуаций

Иногда, чтобы решить задачу, полезно присвоить одним переменным экстремальные значения, а другие переменные сохранить постоянными. Если на переменные не налагаются какие-либо ограничения, то экстремальный сценарий может дать полезные результаты. Большинство из нас подсознательно использует эту стратегию в реальной жизни. Мы, например, спрашиваем себя: «Что может произойти в самом плохом случае?» Определение «наихудшего сценария» — это образчик использования стратегии поиска экстремальной ситуации, которая иногда помогает очень изящно решить проблему. Допустим, вас просят протестировать новый продукт, скажем, хозяйственное мыло. Вам необходимо испытать его в очень холодной и в очень горячей воде, т. е. рассмотреть две экстремальные ситуации, чтобы получить значимый результат. Если оно хорошо работает при экстремальных температурах, то должно вести себя так же и при промежуточных температурах.

Бывает, что использование экстремумов для решения задачи противоречит здравому смыслу. Например, когда возникает вопрос, что лучше, бежать под дождем, чтобы добраться из точки А в точку В, или двигаться медленно, мы вспоминаем, что при быстрой езде на автомобиле под дождем ветровое стекло заливает водой, а при более медленном движении потоки воды не такие сильные. Так что лучше, бежать во время дождя или нет? Анализ экстремальных ситуаций показывает, что очень медленное движение увеличивает время, которое мы находимся под дождем, а экстремально медленное движение, скажем, с нулевой скоростью, приведет к тому, что вы промокнете до нитки. Таким образом, чем быстрее мы будем двигаться, тем меньше намокнем. Вот так экстремумы помогают решать задачи.

Рассмотрим задачу, где стратегия анализа экстремумов помогает найти решение.

В 40 почтовых ящиков в местном почтовом отделении каждое утро кладут письма. Однажды почтальон разложил по этим ящикам 121 письмо. Закончив работу, он обнаружил, что в одном ящике больше писем, чем в любом другом. Какое наименьшее количество писем может находиться в этом ящике?

Поскольку в задаче требуется найти наименьшее количество писем в ящике, мы можем рассмотреть следующую экстремальную ситуацию. Распределим письма равномерно. Предположим, что во всех ящиках находится одинаковое количество писем. Это экстремальная ситуация, противоположностью которой является ситуация, когда все письма лежат в одном ящике. При равномерном распределении в каждый ящик попадает 3 письма — 120: 40 = 3. Добавление дополнительного письма в один из ящиков доведет количество писем в нем до 4 — это и будет наибольшее число. Таким образом, наименьшее количество писем в почтовом ящике, которое превышает количество писем в любом другом ящике, равно 4.

Чтобы попрактиковаться в применении этого метода, рассмотрим еще одну задачу, на этот раз со статистическим уклоном:

Кларисса написала 5 целых чисел. Как оказалось, их мода равна 12, а медианное значение — 14. Среднее арифметическое (или просто среднее) этих чисел равно 16. Одно из чисел больше медианного значения на 5. Какие именно числа написала Кларисса?

Воспользуемся стратегией анализа экстремальной ситуации. Поскольку мода равна 12, наихудший сценарий (наименьшее значение) — это два раза по 12. Мы знаем также, что медиана, или среднее значение равно 14. Так как одно число больше медианы на 5, оно составляет 14 + 5, или 19. Итак, нам известны следующие числа:

12, 12, 14, 19.

Среднее находится путем сложения всех пяти чисел и деления суммы на 5. Поскольку среднее равно 16, то сумма всех чисел составляет 16 x 5 = 80. Найдем сумму уже известных чисел 12 + 12 + 14 + 19 = 57. Недостающее число должно быть равным 80–57 = 23. Таким образом, Кларисса написала следующие числа: 12, 12, 14, 19, 23. Обратите внимание, насколько важно было начать решение с анализа экстремальной ситуации, который позволил определить, что числовой ряд включает в себя два числа 12.

При применении данной стратегии следует, однако, соблюдать осторожность. Анализируя экстремальную ситуацию, следите за тем, чтобы не изменить переменную, которая влияет на другие переменные, а также не изменить сам характер задачи. Задачи, представленные в этой главе, помогают понять, в каких ситуациях можно использовать рассматриваемую стратегию.

Задача 5.1

Автомобиль едет по шоссе с постоянной скоростью 55 км/ч. Водитель замечает другой автомобиль на расстоянии позади. Второй автомобиль обгоняет первый 1 минуту спустя. С какой скоростью двигался второй автомобиль, если считать, что она была постоянной?

Обычный подход

Традиционное решение заключается в составлении таблицы «скорость x время = расстояние», как рекомендуют многие учебные пособия. Это делается следующим образом:



Второй автомобиль ехал со скоростью 85 км/ч.

Образцовое решение

В качестве альтернативы используем стратегию анализа экстремумов. Предположим, что первый автомобиль движется чрезвычайно медленно, т. е. со скоростью 0 км/ч. В этом случае второй автомобиль проедет за одну минуту и догонит первый автомобиль. Таким образом, второй автомобиль должен двигаться со скоростью 30 км/ч. Первый автомобиль движется со скоростью 0 км/ч, а второй автомобиль — на 30 км/ч быстрее, чем первый. Если первый автомобиль будет иметь скорость 55 км/ч, то второй — 85 км/ч.

Задача 5.2

Даны два параллелограмма ABCD и APQR с точкой P на стороне BC и точкой D на стороне RQ, как показано на рис. 5.1. Если площадь параллелограмма ABCD 18, то чему равна площадь параллелограмма APQR?


Обычный подход

Эта задача не такая уж простая. Первая попытка решить ее заключается в поиске признаков конгруэнтности, означающих равенство площадей. Этот метод не дает результата. Более разумно, хотя и не слишком оригинально, провести линию PD, как показано на рис. 5.2.



Теперь можно показать, что площадь треугольника APD составляет половину площади каждого из двух параллелограммов, поскольку в этот треугольник имеет общее основание с обеими параллелограммами и одинаковую высоту. Хотя это разумный подход к решению довольно сложной задачи, существует более изящный способ ее решения.

Образцовое решение

В условиях задачи просто говорится, что точка P лежит на стороне BC, но не указывается, где именно. Мы можем проанализировать экстремальную ситуацию. Можно, например, представить, что точка P совпадает с точкой B. Аналогичным образом можно представить, что точка D, лежащая на стороне RQ, совпадает с точкой R. В результате такого изменения, которое определенно соответствует исходным условиям задачи, два параллелограмма оказываются наложенными друг на друга и, следовательно, имеющими одну и ту же площадь. Таким образом, площадь параллелограмма APRQ равна 18.

Задача 5.3

Суммарное расстояние между съездами 1 и 20 на новой автомагистрали составляет 140 км. Между любыми двумя съездами должно быть не менее 7 км. Чему равно максимальное расстояние между любыми двумя соседними съездами?

Обычный подход

Обычно пытаются подобрать различные комбинации чисел в надежде найти максимум. Существует, однако, более интересный подход.

Образцовое решение

Воспользуемся стратегией анализа экстремальных ситуаций. Прежде всего отметим, что между съездами 1 и 20 всего 19 «расстояний». Поскольку минимальное расстояние между любыми двумя съездами равно 7 км, рассмотрим экстремальную ситуацию, в которой все расстояния, кроме одного, равны 7 км. Тогда минимальная сумма 18 «расстояний» составит 18 x 7 = 126 км. Таким образом, максимальное расстояние между любыми двумя съездами равно 140–126 = 14 км, иначе не хватит километров, чтобы выдержать 7-километровую дистанцию между остальными съездами.

Задача 5.4

У нас есть две однолитровые бутылки. В одной — пол-литра красного вина, в другой — пол-литра белого. Мы берем столовую ложку красного вина, выливаем его в бутылку с белым вином и взбалтываем смесь. Затем мы берем столовую ложку полученной смеси (красного и белого вина) и выливаем ее в бутылку с красным вином.

Чего больше, красного вина в бутылке с белым вином или белого вина в бутылке с красным вином?

Обычный подход

Существует несколько общепринятых подходов к решению такой задачи, в которых используют полученную из условий информацию, например о столовой ложке, которая может быть излишней. При определенном везении и сообразительности можно получить правильный ответ, однако это дело нелегкое, да и ответ нередко кажется неубедительным.

Образцовое решение

Понятно, что размер ложки не имеет реального значения, и что ложки могут быть как большими, так и маленькими. Допустим, мы используем очень большую столовую ложку, такую, которая вмещает пол-литра жидкости, — это будет экстремальная ситуация. После выливания пол-литра красного вина в бутылку с белым вином смесь будет состоять на 50 % из красного вина и на 50 % из белого. Перемешав смесь, мы берем пол-литровую ложку, наполняем ее и возвращаем смесь обратно в бутылку с красным вином. Смесь теперь одинакова в обеих бутылках. Это и есть наш ответ — красного вина в бутылке с белым вином столько же, сколько белого вина в бутылке с красным вином.

Задача 5.5

Найдите недостающие цифры в следующем семизначном числе, которое равно произведению трех последовательных чисел. Чему равны эти три числа? 1 2_ _ _ _6.

Обычный подход

Можно просто попытаться угадать, подставляя различные цифры в надежде, что среди них окажутся искомые. Это крайне маловероятно, хотя и возможно.

Образцовое решение

Вместо догадок воспользуемся стратегией анализа экстремальных ситуаций. Наименьшее возможное число равно 1 200 006, а наибольшее — 1 299 996. Поскольку нам нужен ответ, представляющий собой произведение трех последовательных чисел, проанализируем кубические корни из этих экстремумов, чтобы определить примерную величину этих трех чисел.

Кубический корень из 1 200 006 равен примерно 106, а из 1 299 996 — примерно 109. Это значительно ограничивает простор для выбора. Кроме того, в заданном числе в разряде единиц стоит 6. Значит, три искомые последовательные числа должны оканчиваться либо на 1, 2 и 3, либо на 6, 7 и 8, поскольку их произведения дают 6 в разряде единиц. Имея две такие подсказки, не сложно определить, что искомыми числами будут 106, 107 и 108. Их произведение равно 1 224 936. Задача решена.

Задача 5.6

На рис. 5.3 представлен прямоугольник ABCD со сторонами длиной 8 см и 12 см. Найдите площадь закрашенной области прямоугольника.


Обычный подход

Обычно на задачу смотрят с другой точки зрения и вместо определения площади закрашенной области, найти которую требуется по условиям, определяют площадь незакрашенной области и вычитают ее из площади прямоугольника. Незакрашенный треугольник с основанием AB = 12 см и высотой BC = 8 см, имеет площадь Площадь прямоугольника — это 12 x 8 = 96 см2. Таким образом, площадь закрашенной области равна 96–48 = 48 см2.

Образцовое решение

Другой подход с использованием той же стратегии выглядит следующим образом. Поскольку точное положение точки E не определено, рассмотрим экстремальный случай, когда точка E совпадает с точкой C, как показано на рис. 5.4.

AC — диагональ прямоугольника, которая делит его пополам. Таким образом, закрашенная область занимает точно половину площади прямоугольника, и ее площадь равна 48 см2.

Следует заметить, что тот же прием можно использовать и при замене прямоугольника ABCD на параллелограмм. В первый момент такая задача может показаться сложной, однако она решается аналогичным образом.


Задача 5.7

В офисе директора средней школы им. Джорджа Вашингтона висят 50 почтовых ящиков для учителей. Однажды почтальон принес 151 письмо для учителей. Какое наибольшее число писем может гарантированно получит каждый из учителей?

Обычный подход

Нередко человек, столкнувшись с задачей такого рода, действует наугад и не знает с чего начать. Иногда результат приносит метод проб и ошибок, но убедительного решения он точно не дает.

Образцовое решение

Для решения задач такого рода рекомендуется применять анализ экстремумов. Понятно, что один учитель может получить все письма, однако это не гарантировано. Более реальную оценку ситуации дает экстремальная ситуация, когда письма распределяются предельно равномерно. В этом случае каждый учитель получит по 3 письма за исключением одного, которому попадет еще 151-е письмо. Таким образом, четыре письма — это наибольшее из того, что один учитель может гарантированно получить.

Задача 5.8

Точка M лежит на середине стороны AB ?ABC. Точка P может находиться в любом месте на отрезке AM (рис. 5.5). Линия, проведенная через точку M параллельно PC, пересекается с BC в точке D. Какую часть площади ?ABC составляет площадь ?BDP?

Обычный подход

Площадь ?BMC равна половине площади ?ABC (в силу того, что медиана делит треугольник на две равные части). Площадь ?BMC = площадь ?BMD + площадь ?CMD = площадь ?BMD + площадь ?MPD, которая равна площади площади ?ABC. Это следует из того, что треугольники, вершины которых лежат на линии, параллельной общему основанию, имеют равные площади.


Образцовое решение

Решение этой задачи значительно упрощается при использовании стратегии анализа экстремальных ситуаций. Поместим точку P в экстремальную позицию так, чтобы она совпадала с точкой M или точкой A. Допустим, точка P совпадает с точкой A. Обратите внимание на то, что по мере смещения точки P вдоль BA в направлении точки A линия MD, которая должна оставаться параллельной PC, смещается так, что D приближается к средней точке линии BC. В конечном положении точки D линия AD становится медианой ?ABC. Поскольку медиана делит треугольник на два треугольника с равными площадями, площадь ?PBD равна половине площади ?ABC.

Данное решение с помощью стратегии анализа экстремальных ситуаций ясно показывает важность отслеживания всех перемещений по мере смещения точки в предельное положение.

Задача 5.9

Два конгруэнтных квадрата, длина стороны которых равна 4 см, размещены так, что вершина одного из них находится в центре другого. Чему равно наименьшее значение площади пересекающейся части (рис. 5.6)?

Обычный подход

Наиболее очевидный прием — построить два квадрата. Некоторые даже вычерчивают их в масштабе и пытаются измерить искомую площадь. Поскольку фигура получается неправильной, измерение ее площади может оказаться сложным.



Другой подход — провести несколько вспомогательных линий, например линии BM и CM. Несложно доказать, что два треугольника BSM и CTM конгруэнтны (конгруэнтность по двум углам и стороне) (см. рис. 5.7). Четырехугольник SCTM равен по площади треугольнику BCM, поскольку площадь треугольника добавляется к площади двух треугольников, которые, как мы доказали, являются конгруэнтными.

Образцовое решение

Поскольку ориентация квадратов не определена в условиях задачи, их можно разместить так, как нам захочется, лишь бы вершина одного находилась в центре другого. Обратимся к нашей стратегии анализа экстремальных ситуаций. Можно разместить квадраты так, как показано на рис. 5.8, где стороны этих фигур взаимно перпендикулярны.



Если этого недостаточно, чтобы удостовериться в равенстве закрашенной области четверти исходного квадрата, то нужно лишь продолжить линии PM и NM до пересечения со сторонами квадрата в точках J и K соответственно, как показано на рис. 5.9.

Очевидно, что закрашенная область равна площади квадрата, или части 16 см2, т. е. 4 см2. Разместив квадраты в определенном положении, мы легко нашли ответ.



Задача 5.10

Найдите значение x, которое удовлетворяет уравнению:

Обычный подход

На первый взгляд задача кажется настолько ошеломляющей, что большинство людей не знают, как к ней подойти. И это не удивительно.

Образцовое решение

Посмотрим на это, как на своего рода экстремальную ситуацию. Начнем с того, что количество x в этом ряду бесконечно. Отбрасывание одной неизвестной x не должно никак влиять на результат в силу характера бесконечности. Таким образом, удалив первую неизвестную x, мы обнаружим, что все оставшиеся x так же должны быть равны 2. Это позволяет переписать уравнение, как x2 = 2. Следовательно x = ±?2. Если ограничиться положительными числами, то ответом будет x = ?2.

Ниже показано, как с увеличением ряда значение приближается к 2.



Вот мы и нашли удивительно простое решение для очень сложной на первой взгляд задачи.

Задача 5.11

В одной из старейших телевизионных игр под названием Let's Make a Deal участникам предлагают сделать слепой выбор. Случайно выбранный зритель выходит на сцену и видит перед собой три двери, за одной из которых находится автомобиль, а за двумя другими — ослы. Участник надеется выбрать ту дверь, за которой спрятан автомобиль, а не ту, где стоит осел. Если он угадывает, то получает автомобиль в качестве приза. Однако не все так просто. После того, как участник сделает выбор, ведущий, Монти Холл, который знает, за какой дверью находится автомобиль, показывает одного из ослов (не открывая две оставшиеся двери) и спрашивает участника, хочет ли он оставить первоначальный выбор (что кроется за ним пока неизвестно) или, может быть, передумает и выберет другую неоткрытую дверь. В этот момент, чтобы повысить напряжение, публика начинает скандировать «оставить» или «передумать». Вопрос в том, что делать? Есть ли разница? Если так, то какую стратегию лучше использовать (т. е. что может повысить вероятность выигрыша)?

Обычный подход

Кто-то скажет, что лучше всего принимать решение интуитивно. Большинство, однако, придерживаются мнения, что в любом случае разницы никакой нет, поскольку у вас один шанс из двух получить автомобиль. И те и другие неправы, а раз так, то есть серьезный мотив, разобраться что к чему.

Образцовое решение

Лучше всего рассмотреть эту задачу шаг за шагом, а потом, для уверенности, рассмотреть экстремальную ситуацию и принять решение.



Разберем задачу шаг за шагом. Результат постепенно станет ясным. За дверями находятся два осла и один автомобиль. Вы должны попытаться получить автомобиль. Вы выбираете дверь 3. Монти Холл открывает одну из дверей, которую вы не выбрали, и показывает осла.



Он спрашивает: «Вы все еще хотите оставить свой первоначальный выбор или передумаете и выберете другую закрытую дверь?» Чтобы помочь принять решение, воспользуемся стратегией анализа экстремальных ситуаций. Допустим, у нас 1000 дверей, а не три.



Вы выбираете дверь 1000. Какова вероятность того, что это правильная дверь?

«Очень маловероятно», поскольку вероятность выбора правильной двери равна

Насколько вероятно, что автомобиль находится за одной из других дверей (1–999)?

«Очень вероятно»:



Это все «очень вероятные» двери!



Теперь мы готовы ответить на вопрос. Какой выбор будет наилучшим?

• Дверь 1000 («очень маловероятная» дверь) или

• Дверь 1 («очень вероятная» дверь)?


Ответ теперь очевиден. Мы должны выбрать «очень вероятную» дверь, т. е. «передумать» — это лучшая стратегия для участника шоу. В экстремальной ситуации намного легче увидеть лучшую стратегию, чем при анализе ситуации с тремя дверями, как в исходных условиях задачи. Принцип одинаков в любой ситуации.

Эта задача вызвала немало споров в научных кругах и даже попала на страницы газеты The New York Times и других популярных изданий. Джон Тирни написал в The New York Times (Sunday, July 21, 1991), что «возможно это только иллюзия, однако похоже спору, в котором участвовали все от математиков до читателей журнала Parade и любителей телеигры Let's Make a Deal, положен конец. Спор начался, когда Мэрилин вос Савант опубликовала головоломку в журнале Parade. Как известно читателям ее колонки „Спросите Мэрилин“, имя г-жи вос Савант включено в списки Галереи славы Книги рекордов Гиннесса за обладание „наивысшего IQ“. Но этот факт производит на публику не такое впечатление, как то, что она сумела ответить на вопрос читателя». Мэрилин дала правильный ответ, и, хотя многие математики продолжают спорить, мы решили эту задачу!

Глава 6
Решение более простой аналогичной задачи

Некоторые задачи на первый взгляд кажутся чрезвычайно сложными. Смутить могут, например, очень большие числа. А иногда в замешательство приводит излишнее количество данных, некоторые из которых совершенно не нужны для решения. Даже формулировка задачи способна поставить в тупик. Что бы там ни было, отличный подход — упростить задачу, однако так, чтобы она осталась эквивалентной исходному варианту. Попробуйте уменьшить числа, изменить рисунок или как-то иначе упростить задачу. Решая упрощенную версию, вы получаете представление о том, как справиться с исходной задачей.

Купив первый раз новый компьютер, вы ведь не брались за освоение сразу всех функций и возможностей, а, скорее всего, начинали с чего-то более знакомого и постепенно добавляли новое, пока не выясняли, на что способно приобретение. Наверняка вы брались сначала за более простые функции.

Допустим, вам нужно решить следующую задачу:

Дано 19 последовательных целых чисел, сумма которых равна 95. Какое число стоит на десятом месте в этом ряду?

Одни, скорее всего, попробуют применить алгебраический подход и представят 19 целых чисел, как x, (x + 1), (x + 2), (x + 3), …, (x + 17), (x + 18) и так далее. Так они дойдут до 95, а потом найдут решение для x. Другие заметят, что число в десятой позиции является средним, обозначат его как x и представят остальные числа следующим образом: (x + 9), (x + 8), (x + 7), …, (x - 7), (x - 8), (x - 9). Теперь можно объединять парные члены по мере их добавления, т. е. преобразовывать (x - 9) и (x + 9) в 2x, (x - 8) и (x + 8) тоже в 2x и т. д., получая каждый раз 2x. Такая версия намного проще для решения, поскольку мы получаем уравнение вида 9(2x) + x = 95, или 19x = 95 откуда x = 5.

Вместе с тем есть еще более интересный подход. Допустим, мы рассматриваем более короткий ряд чисел, например 3 + 4 + 5 + 6 + 7. Их сумма (25) при делении на 5 дает среднее, а именно 5, которое оказывается средним числом ряда. Для ряда в нашей задаче средним число является 10-й член, а поскольку целые числа последовательны, этот член является также средним арифметическим, или средним числового ряда из 19 членов. Таким образом, чтобы найти среднее, нужно просто взять сумму (95) и разделить ее на количество членов ряда (19). Ответ — 5. Эта упрощенная версия задачи позволяет представить исходное задание в значительно более простой форме и, таким образом, облегчить решение.

Зачастую можно не ограничиваться простым уменьшением сложности исходной задачи, а применить также и другие наши стратегии. Например, найдите десятичное значение числа 1/500 000 000 000.

Воспользоваться калькулятором здесь не удастся, поскольку дисплеи большинства из них не воспроизводят 12-значные числа. Применим две другие стратегии: организуем данные и найдем закономерность. Решим ряд более простых версий нашей задачи, представим результаты в табличной форме, а потом посмотрим, нет ли в них какой-либо закономерности.



Здесь определенно просматривается закономерность. Количество нулей в знаменателе равно количеству нулей между запятой и 2. Поскольку в знаменателе 11 нулей после 5, между запятой и 2 должно быть тоже 11 нулей:



Обратите внимание, насколько упрощенная версия(и) исходной задачи вместе с двумя другими стратегиями облегчают решение. Имейте в виду, что использование нескольких стратегий для решения задачи не редкость.

Задача 6.1

Баскетбольная команда принимает участие в конкурсе на лучшее исполнение штрафных бросков. Первый игрок успешно выполняет x штрафных бросков, второй — y, а третий — количество бросков, равное среднему арифметическому количества бросков первых двух игроков. Каждый последующий игрок успешно выполняет такое количество бросков, которое равно среднему арифметическому бросков всех предыдущих игроков. Сколько успешных штрафных бросков сделает 12-й игрок?

Обычный подход

Некоторые пытаются решить такую задачу через определение среднеарифметического значения для каждого из 12 игроков по очереди. На это нужно много времени и сил. К тому же очень легко сделать ошибку при вычислениях. У задачи наверняка должно существовать более рациональное решение.

Образцовое решение

Мы начнем с анализа более простой аналогичной задачи. Заменим x и y простыми числами и посмотрим, что происходит. Допустим, первый игрок сделал 8 штрафных бросков (x), а второй — 12 (y). Тогда счет третьего игрока будет равен их среднему арифметическому, т. е. Четвертый игрок наберет среднее арифметическое бросков первых трех игроков, т. е. а пятый — среднее арифметическое бросков первых четырех игроков, т. е. Ну вот! Счет любого игрока после первых двух всегда равен среднему арифметическому успешных бросков первых двух игроков. Правильным ответом на эту задачу будет среднее арифметическое успешных бросков первых двух игроков, а именно Упрощенная аналогичная задача позволила нам определить метод, который нужно использовать для быстрого решения исходной задачи.

Задача 6.2

Сумма расстояний от любой точки внутри или на сторонах равностороннего треугольника до трех сторон всегда постоянна. Чему равна сумма этих расстояний, если сторона равностороннего треугольника равна 4?

Обычный подход

Существуют несколько способов решения этой задачи. Один из наиболее простых способов — выбрать какую-нибудь точку внутри равностороннего треугольника (т. е. сделать нечто вполне ожидаемое) и провести из нее три перпендикуляра к сторонам (рис. 6.1).



Приравняв площадь ?ABC и сумму площадей треугольников APB, PBC и CPA при использовании трех высот x, y, z и основания 4, мы получим площадь:



Таким образом, h = x + y + z. В нашем случае высота равностороннего треугольника равна 2 ?3. Значит x + y + z = 2 ?3.

Образцовое решение

Без ущерба общему смыслу задачи рассмотрим более простой аналогичный пример, поскольку мы вправе поместить точку P в любом месте внутри равностороннего треугольника или на его сторонах. Если совместить точку P с точкой A, то решение становится очевидным. Перпендикуляры к сторонам AB и AC имеют длину 0, а перпендикуляр к стороне BC — это просто высота треугольника, или 2 ?3. Обратите внимание на то, что такую стратегию можно также классифицировать, как анализ экстремальных ситуаций. Мы рассмотрели экстремальную ситуацию, в которой точка совмещена с вершиной треугольника. Это лишний раз подчеркивает гибкость выбора стратегии.

Задача 6.3

В приведенных ниже выражениях m и n — положительные целые числа, каждое из которых больше 1. Какое из выражений имеет наибольшее значение?


Обычный подход

Наиболее очевидный подход — реально выполнить операции как есть и попытаться выяснить, какое из выражений имеет наибольшее значение. Это громоздкий и нудный метод, требующий, к тому же, большого объема вычислений.

Образцовое решение

Попробуем решить более простую версию этой задачи. Для ее упрощения подставим вместо переменных подходящие положительные целые числа. Пусть m = 2, а n = 4. Тогда выражение (1) будет равно 2 + 4 = 6; выражение (2) — 2–4 = –2; выражение (3) — ?16 = 4; выражение выражение Из этого следует, что наибольшее значение имеет выражение m + n.

Задача 6.4


Обычный подход

Традиционный подход заключается в решении уравнения и определении значения x, которое равно Затем это значение подставляют в выражение и получают Конечно, это связано с определенными алгебраическими и арифметическими преобразованиями, однако в конечном итоге дает правильный ответ.

Образцовое решение

Лучше, однако, взглянуть на задачу с другой точки зрения, начиная с исходной информации: уравнения Если взять обратные величины обеих сторон уравнения, мы получим уравнение вида которое намного легче поддается решению. Поскольку нужно найти значение x + 6, мы просто прибавим 1 к обеим частям этого уравнения и получим или Возьмем опять обратные величины обеих сторон уравнения и получим, что и требовалось найти. Это несомненно более изящный подход.

Задача 6.5

Дан круг и его диаметр; покажите, как разделить площадь на семь частей равной площади без использования прямых линий.

Обычный подход

Обычно при виде такой задачи человек понимает, что циркуль — это то, что надо, и начинает чертить окружности внутри исходного круга в надежде обнаружить какую-либо закономерность. Чаще всего такое упражнение не дает ничего.

Образцовое решение

Возьмем наш круг и отложим от одного края диаметра отрезок, равный одной седьмой части его длины, как показано на рис. 6.2.



Площадь более светлой закрашенной области можно описать, как площадь половины исходного круга плюс площадь полукруга X минус площадь полукруга Y.

Известно, что отношение площадей кругов прямо пропорционально отношению квадратов их диаметров, поэтому площадь более светлой закрашенной области можно представить следующим образом:

Площадь (X + Z) = площадь (Y + Z) — площадь (Y).

Поскольку у трех полусфер отношение диаметров составляет (Y + Z):(Y):(X) = 7:6:1, отношение их площадей равно 49:36:1. Используя это, можно увидеть, что отношение площади более светлой закрашенной области к площади большого полукруга составляет (49–36 + 1):49 (или 14:49), иначе говоря, площадь более светлой закрашенной области равна площади большой полусферы. В этом случае отношение площади более светлой закрашенной области к площади целого круга равно Мы умножаем на потому, что представляет собой отношение к площади круга. Используя эту стратегию, мы можем рассмотреть полукруги с диаметрами AC, AD, AE, AF, AG и AH, которые делят площадь круга на семь частей равной площади.

Задача 6.6

Два поезда, один из Чикаго в Нью-Йорк, а другой из Нью-Йорка в Чикаго (расстояние 800 км) одновременно выходят навстречу друг другу по одной колее и идут с постоянной скоростью 60 и 40 км/ч соответственно. Перед одним из поездов летит пчела со скоростью 80 км/ч. После достижения идущего навстречу поезда пчела разворачивается и летит обратно (все с той же скоростью 80 км/ч). Пчела летает туда-обратно до тех пор, пока поезда не сталкиваются и не сплющивают ее в лепешку. Сколько километров пролетает пчела?

Обычный подход

Эта задача может напомнить читателю известный пример, приводимый в большинстве учебников алгебры, однако в ней есть необычный момент, отсутствующий в подобных задачах на равномерное движение. Естественно, возникает желание определить отдельные расстояния, которые пролетала пчела. Первой реакцией является составление уравнения на основе знакомой формулы: «скорость, умноженная на время, дает расстояние». Однако определение этого пути туда-обратно довольно сложное дело и связано с большим объемом вычислений. В любом случае, решить задачу подобным образом очень сложно.

Образцовое решение

Значительно более изящный подход предполагает решение упрощенной аналогичной задачи (можно сказать также, что это подход к решению с другой точки зрения). Мы ищем расстояние, которое пролетела пчела. Если знать время, в течение которого летала пчела, то определить пройденное расстояние будет легко, поскольку скорость пчелы известна.

Время полета пчелы узнать несложно, так как оно равно времени движения поездов до столкновения. Для определения времени t движения поездов составим следующие уравнения.

Расстояние, пройденное первым поездом равно 60t, а второго — 40t. Суммарное расстояние, пройденное поездами, составляет 800 км. Таким образом, 60t + 40t = 800, а t = 8 часам. Иначе говоря, пчела летала 8 часов. Теперь можно найти расстояние, которое пролетела пчела: 8 x 80 = 640 км. Внешне невероятно трудное задание определить расстояние, пройденное летающей туда-сюда пчелы, было сведено к довольно обычной задаче «на равномерное движение», решение которой очевидно.

Задача 6.7

Имеется произвольно начерченная пентаграмма, показанная на рис. 6.3. Определите, чему равна сумма острых углов при ее вершинах.


Обычный подход

Большинство, к сожалению, пытается измерить углы с помощью транспортира (надо надеяться, с достаточной точностью). На основании полученного результата строятся предположения о том, какой должна быть эта сумма.

Образцовое решение

Мы же воспользуемся стратегией решения упрощенной аналогичной задачи. Иначе говоря, поскольку форма, или правильность не определена, предположим, что это пентаграмма, вписанная в окружность, как показано на рис. 6.4. Если посмотреть на острые углы пентаграммы, можно заметить, что каждый из них является вписанным в окружность углом, равным по определению половине дуги, на который он опирается. Например, Глядя на дуги оставшихся четырех острых углов пентаграммы, видно, что в сумме они составляют полную окружность. Итак, мы знаем, что сумма углов равна половине суммы дуг, на которые они опираются, т. е. она равна половине окружности, или 180?.


Задача 6.8

Какое из следующих чисел имеет наибольшее значение?

148, 242, 336, 430, 524, 618, 712, 86

Обычный подход

С помощью компьютерной программы или даже калькулятора, который может оперировать большими числами, можно попытаться реально определить значение каждого числа. Однако такой подход утомителен и требует много времени. Тем не менее он имеет право на существование.

Образцовое решение

Воспользуемся стратегией решения более простой аналогичной задачи. Даже при быстром взгляде на числа видно, что показатели степени кратны 6. Если извлечь корень шестой степени из каждого члена ряда (или возвести его в степень), то можно упростить сравнение. Иначе говоря, мы знаем, что все исходные числа являются производными 6-й степени. Таким образом, наибольшее значение в следующем ряду будет связано с наибольшим значением исходных чисел, которые требуется сравнить.

18, 27, 36, 45, 54, 63, 72, 81.

Значения чисел в этом ряду определить несложно:

27 = 128; 36 = 729; 45 = 1024; 54 = 625; 63 = 216.

Остальные числа явно меньше. Итак, наибольшее значение в ряду из восьми чисел, возведенных в степень, имеет 430, которое можно представить как (45)6.

Задача 6.9

Чтобы растянуть удовольствие от бутылки вина объемом 16 унций, Дэвид придумал следующее. В первый день он выпивает только 1 унцию вина и доливает в бутылку столько же воды. Во второй день он выпивает 2 унции смеси вина с водой и опять доливает в бутылку столько же воды. На третий день он выпивает 3 унции смеси вина с водой и вновь доливает в бутылку столько же воды. Процесс продолжается до тех пор, пока на 16 день Дэвид не опорожняет всю бутылку объемом 16 унций. Сколько всего унций воды выпил Дэвид?

Обычный подход

В задаче вроде этой очень легко утонуть в деталях. Некоторые, наверное, уже составляют таблицу, вносят в нее данные об объеме вина и воды в бутылке каждый день и пытаются вычислить пропорциональные количества той и другой жидкости, выпиваемой Дэвидом каждый день. Задачу легче решить, взглянув на нее с другой точки зрения, а именно, задавшись вопросом, сколько воды Дэвид добавляет в смесь каждый день. Поскольку он в конечном итоге опорожняет бутылку (на 16-й день), и в ней ничего не остается, Дэвид, надо полагать, выпивает всю долитую воду. В первый день он долил 1 унцию воды, во второй — 2 унции, в третий — 3 унции. На 15-й день в бутылку было добавлено 15 унций воды. (Не забывайте, что в 16-й день вода не добавлялась.) Таким образом, количество воды, выпитой Дэвидом, равно 1 + 2 + 3 + 4 + 5 + 6 + 7 + 8 + 9 + 10 + 11 + 12 + 13 + 14 + 15 = 120 унциям.

Образцовое решение

Хотя приведенное выше решение имеет право на существование, можно рассмотреть чуть более простую аналогичную задачу и определить, сколько жидкости Дэвид выпил в целом, а потом просто вычесть из результата объем вина, т. е. 16 унций.

Таким образом, 1 + 2 + 3 + 4 + 5 + 6 + 7 + 8 + 9 + 10 + 11 + 12 + 13 + 14 + 15 + 16 = 136, и 136 - 16 = 120.

Дэвид выпил 136 унций жидкости, из которой 120 унций приходилось на воду.

Глава 7
Организация данных

Одной из наиболее важных стратегий является организация данных, хотя, на первый взгляд, это понятно и без слов. Иначе говоря, все должны автоматически упорядочивать данные из условий задачи. В жизни мы делаем это подсознательно каждый день.

Так, приступая к заполнению налоговой декларации каждую весну, мы автоматически организуем данные в определенном порядке без всяких подсказок. От того, как будут организованы квитанции, чеки, формы W-2 и т. п., сильно зависит заполнение сложных форм налоговой отчетности.

Большинство людей составляют тщательно организованный список покупок, прежде чем отправиться в супермаркет. Они могут разбивать покупки по категориям, по месту расположения в магазине или по степени необходимости. Аналогичным образом, отправляясь в турпоездку, мы чаще всего составляем перечень того, что хотим посмотреть. При этом список можно изложить на бумаге или просто держать в голове.

Когда крупные организации проводят опросы, нет ничего необычного в том, что они получают разные результаты, — все зависит от того, как в каждом случае организуются одни и те же данные.

В задачах, включающих множество данных, людей нередко приводит в замешательство представление этих данных. Умение организовывать данные в логичной и ясной форме очень помогает решению задач. Рассмотрим одну из задач, где применяется эта стратегия.

Группа археологов ведет раскопки. Они ежедневно на протяжении 15 дней находят множество осколков глиняной посуды:

13, 45, 12, 47, 8, 18, 13, 27, 98, 11, 23, 67, 51, 14, 6.

Чему равно медианное количество найденных ими осколков глиняной посуды?

При таком представлении количества ежедневных находок решить данную задачу практически невозможно. Организуем данные более логичным образом — в порядке возрастания:

6, 8, 11, 12, 13, 14, 18, 23, 27, 45, 47, 51, 69, 98.

Теперь найти медиану не представляет труда. Это среднее число, в нашем случае восьмое, или 18.

Рассмотрим еще одну задачу, где очень важна организация данных.

Джек и Марлин хотят вступить в клуб любителей кино на DVD. Они получают два предложения. Клуб Freedom Movie взимает вступительный взнос в размере $20, а потом берет $6,20 за каждый фильм. Клуб New Look не требует вступительного взноса, однако берет $8,10 за каждый DVD. Джек решает присоединиться к клубу Freedom Movie, а Марлин — к клубу New Look. Сколько DVD каждый из них должен купить, прежде чем Марлин истратит больше Джека? Насколько больше она истратит?

Чтобы решить задачу, организуем данные в три колонки:



Когда они купят по 11 DVD, Марлин истратит больше денег, чем Джек. Марлин придется заплатить $89,10 - $88,20, или на 90 центов больше. Ответы на оба вопроса легко найти, проанализировав представленные в табличной форме данные.

А вот геометрическая задача, решить которую можно только при тщательной организации данных.

Треугольник имеет сумму сторон и периметр, равные 12. Чему равны длины его трех сторон?

Подготовим и организуем перечень данных, обозначив стороны треугольника, как A, B и C. Начнем с A = 1 и перечисления всех возможностей для A = 1. Затем сделаем то же самое для A = 2 и т. д.



В этом перечне все тройки чисел дают сумму, равную 12. Не забывайте, однако, что в треугольнике сумма любых двух сторон всегда больше третьей стороны, иначе треугольник не может существовать. Это условие исключает большее число сочетаний. Единственные три возможности — это 2–5–5, 4–4–4 и 3–4–5. Представление данных в упорядоченном виде значительно облегчает решение задачи.

В этой главе мы представим задачи, которые наиболее эффективно решаются путем организации данных логичным образом. Хотя некоторые из них можно решить и другими способами, они приводятся для демонстрации преимуществ этого вроде бы необычного метода решения.

Задача 7.1

Между двумя баскетбольными командами устраивают конкурс на лучшее исполнение штрафных бросков. В финал выходят Робби и Сэнди. Победителем становится тот, кто первым выполнит подряд два успешных штрафных броска или в целом три штрафных броска. Сколько комбинаций бросков может привести к победе?

Обычный подход

Большинство начинает с попыток найти все возможные комбинации, которые могут привести к победе. Однако непонятно, как определить, все ли комбинации учтены. Это довольно проблематичная задача.

Образцовое решение

Воспользуемся стратегией организации данных и составим два исчерпывающих перечня путей достижения победы каждым игроком. Первый перечень показывает результаты, когда первый бросок делает Робби, второй — когда первый бросок делает Сэнди.



Существует 10 возможных комбинаций, на которых конкурс завершается. Исчерпывающий перечень комбинаций упорядоченно и наглядно представляет возможности.

Задача 7.2

Сколько треугольников изображено на рис. 7.1?


Обычный подход

Как правило, люди начинают подсчитывать треугольники в том или ином порядке, но без определенной системы. Чаще всего это приводит к путанице и неуверенности в том, все ли треугольники учтены. Другой традиционный подход предполагает использование формальных методов подсчета. В этом случае определяются комбинации, которые могут быть образованы шестью линиями, и исключаются комбинации, образующиеся в результате совпадений. Количество комбинаций из шести линий по три равно 6C3 = 20. Из этого результата нужно вычесть три совпадения (по вершинам). Таким образом, в фигуре на рисунке 17 треугольников.

Образцовое решение

Для упрощения задачи преобразуем фигуру, будем постепенно добавлять линии и считать, что получается в результате использования такой формы организации данных. Иначе говоря, мы будем подсчитывать треугольники, образующиеся при добавлении каждой части. Начнем с исходного треугольника ABC. Итак, в начале мы имеем всего один треугольник.



Теперь рассмотрим треугольник ABC с одной внутренней линией AD. У нас получилось два новых треугольника ABD и ADC.



Добавим еще одну внутреннюю линию BE и подсчитаем все новые треугольники, имеющие сторону BE.



Таким же образом добавим линию CF и опять подсчитаем новые треугольники, имеющие сторону CF.



Представим полученные результаты в табличной форме.



Общее количество перечисленных выше треугольников равно 17.

Задача 7.3

Дана последовательность Найдите положительное целое число n, при котором произведение первых n членов последовательности превышает 100 000.

Обычный подход

В этом случае обычно прибегают к методу проб и ошибок и начинают добавлять члены в последовательность и перемножать их до тех пор, пока произведение не превысит 100 000. Такой подход трудоемок, и его точно нельзя назвать изящным.

Образцовое решение

Напишем сначала произведение первых n членов данной последовательности, что в определенном смысле будет организацией и представлением наших данных в более удобной форме:



«Превышает 100 000» означает, что нам нужно число, большее, чем 105, а это происходит, только когда или n (n + 1) > 110. Когда n <= 10, мы получаем n (n + 1) <= 110. Таким образом, наименьшее целое значение n, при котором выполняется условие задачи, равно 11.

Задача 7.4

Джером открыл свое первое предприятие по прокату каяков. За прокат он берет почасовую оплату. Каякам присваиваются идентификационные номера, на каждом из них стоят три цифры. Первая цифра — это номер предприятия, а именно 1. Номера у каяков не могут повторяться, а три цифры должны располагаться в возрастающем порядке. Ноль использовать нельзя. Вскоре Джером обнаружил, что использовал все возможные сочетания, которые удовлетворяют условиям. Какое максимальное количество каяков может быть у Джерома?

Обычный подход

Самый распространенный подход — выписывание всех возможных трехзначных чисел, удовлетворяющих условиям задачи. Но как узнать, все ли эти числа учтены? Существует ли метод, обеспечивающий гарантированное решение? Обычный подход явно не самый эффективный!

Образцовое решение

Представим наши данные в табличной форме:



Джером может иметь не более чем 7 + 6 + 5 + 4 + 3 + 2 + 1 = 28 каяков.

Задача 7.5

Фермер везет яблоки на рынок. Яблоки уложены в шесть ящиков. Весы на пункте взвешивания могут принять за раз только пять ящиков. Нам дают результаты шести взвешиваний:

Ящик B + ящик C + ящик D + ящик E + ящик F = 200 фунтов;

Ящик A + ящик C + ящик D + ящик E + ящик F = 220 фунтов;

Ящик A + ящик B + ящик D + ящик E + ящик F = 240 фунтов;

Ящик A + ящик B + ящик C + ящик E + ящик F = 260 фунтов;

Ящик A + ящик B + ящик C + ящик D + ящик F = 280 фунтов;

Ящик A + ящик B + ящик C + ящик D + ящик E = 300 фунтов.

Сколько фунтов яблок в каждом ящике?

Обычный подход

Эту задачу можно решить алгебраически, составив шесть уравнений с шестью неизвестными:

B + C + D + E + F = 200;

A + C + D + E + F = 220;

A + B + D + E + F = 240;

A + B + C + E + F = 260;

A + B + C + D + F = 280;

A + B + C + D + E = 300.

Решение шести уравнений довольно трудоемко, поэтому попробуем поискать другой подход к этой задаче.

Образцовое решение

С помощью нашей стратегии организации данных можно упростить решение задачи и сделать его изящным. Начнем с представления данных в табличной форме:



Мы опять получили довольно громоздкий набор уравнений, но можно посмотреть на них с другой точки зрения и организовать данные вертикально, просуммировав колонки в вертикальном направлении:

5A + 5B + 5C + 5D + 5E + 5F = 1500.

Разделив обе стороны уравнения на 5, мы получаем:

A + B + C + D + E + F = 300.

Однако шестое взвешивание в таблице показывает, что A + B + C + D + E = 300 фунтам. Следовательно, ящик F должен весить 0 фунтов. Обратимся затем к пятому взвешиванию, которое показывает, что A + B + C + D + F = 280 фунтам. Однако мы уже знаем, что F = 0, а значит A + B + C + D = 280.

Вернемся к шестому взвешиванию — A + B + C + D + E = 300, вычтем из него последнее уравнение и получим E = 20 фунтов.

Из четвертого взвешивания следует, что A + B + C + E + F = 260. Подставив в это уравнение уже известные значения F и E, мы получим A + B + C + 20 + 0 = 260, или A + B + C = 240. Подставляя это значение в пятое взвешивание, находим D = 40.

Если вычесть уравнение третьего взвешивания из уравнения четвертого взвешивания, то, зная, что F = 0, мы получаем:



Поскольку D = 40, мы получаем C = 60.

Подставим известные значения в уравнение первого взвешивания: B + C + D + E + F = 200 = B + 60 + 40 + 20 + 0. Таким образом, B = 80.

Поступив аналогичным образом с уравнением второго взвешивания, получим A = 100.

Использование табличной формы сделало данные более понятными и позволило решить задачу путем логических рассуждений.

Задача 7.6

Даны трехзначные числа, которые составлены исключительно из нечетных цифр. Чему равна сумма всех этих чисел?

Обычный подход

Обычно при решении задачи такого типа начинают составлять список нечетных чисел в том или ином порядке, а потом долго складывают их.

Образцовое решение

Главное здесь — организовать числа логичным образом. Например, наш список может выглядеть так: 111 + 113 + 115 + 117 + 119 + 133 + 135 + 137 + 139 + … + 511 + 513 + 515 + 517 + 519 + … + 991 + 993 + 995 + 997 + 999. Поскольку всего пять цифр могут находиться в каждом из трех разрядов, существует 5 x 5 x 5 = 125 возможных чисел. Если подойти к делу организованно, то можно складывать эти числа парами: первое и последнее, второе и предпоследнее и т. д. Сумма каждой из этих пар равна 1110. В нашем списке пар чисел. Таким образом, сумма этих чисел составляет

Данные можно организовать по-другому и также получить довольно изящное решение. Мы уже определили, что сложить нужно 125 целых чисел, каждое из которых состоит из трех цифр, а значит всего нам необходимо принять во внимание 375 цифр. Понятно, что каждое из пяти нечетных цифр — 1, 3, 5, 7 и 9 — встречается 75 раз, т. е. 25 раз в каждом разряде (в разряде сотен, десятков и единиц). Это можно представить в виде формулы следующим образом:

25 [100 (1 + 3 + 5 + 7 + 9) + 10 (1 + 3 + 5 + 7 + 9) + 1 (1 + 3 + 5 + 7 + 9)] = 25 x 25 x (100 + 10 + 1) = 69 375.

В каждом из приведенных примеров организации данных решение задачи становится значительно более изящным, чем в случае использования лобового метода.

Задача 7.7

Допустим, у нас есть 11 линий, лежащих в одной плоскости, при этом три линии проходят через точку P, а три линии имеют общую точку Q. Никакие другие три линии, кроме этих, не пересекаются. Чему равно минимальное количество точек пересечения этих 11 линий при таких условиях?

Обычный подход

Чаще всего эту задачу пытаются решить методом проб и ошибок, но довольно большое количество линий (11) делает такой подход проблематичным. Таким образом, должен быть какой-то другой, более эффективный способ решения подобной задачи.

Образцовое решение

Чтобы решить такую задачу, нужно организовать линии логичным образом. Начнем с построения трех линий, пересекающихся в точке P, как показано на рис. 7.6.



Повторим эту процедуру с точкой Q, построив линии l3||l4 и l2||l5, как показано на рис. 7.7.



Затем проведем шесть оставшихся линий параллельно линии l2. Это показано на рис. 7.8. Каждая из этих линий добавляет три новые точки пересечения.



Таким образом, в результате организации исходных данных логичным образом мы получаем следующее количество точек пересечения: 6 x 3 + 4 = 22.

Задача 7.8

Если напечатать все числа от 1 до 1 000 000, сколько раз в них встретится цифра 8?

Обычный подход

Обычно в ответ на такой вопрос, который кажется ошеломляющим, начинают составлять список чисел без какого-либо намека на их организацию. При таком подходе решение зависит от того, удастся ли увидеть какую-нибудь закономерность в числовом ряду.

Образцовое решение

Лучшая стратегия здесь заключается, пожалуй, в организации данных таким образом, чтобы можно было выявить любую закономерность в списке, если она существует.



В представленных выше шести разрядах миллиона цифры 0, 1, 2, 3, 4, …, 8 и 9 используются равное количество раз. Это понятно потому, что каждую «комбинацию» можно составить из каждой из 10 цифр. Таким образом, цифра 8 должна появиться в случаев, или 600 000 раз.

Задача 7.9

У продавца двое часов, которые бьют полночь одновременно. Однако одни часы спешат на 1 минуту в час, а другие отстают на 1 минуту в час. Если такая разница сохранится и дальше, то через какое время их показания совпадут?

Обычный подход

Первая мысль — составить уравнение. Если обозначить за x время, через которое показания часов совпадут, то мы получим 12 + x = 12 — x. Решение этого уравнения дает 2x = 0, и x = 0. От такого решения толку мало.

Образцовое решение

В сутках 24 часа, за это время часы A уйдут вперед на 24 минуты, а часы B отстанут на 24 минуты. Через пять дней часы A уйдут вперед на 5 x 24 = 120 минут, или на 2 часа. В свою очередь часы B будут отставать на 120 минут, или 2 часа каждые пять дней. Воспользуемся нашей стратегией и представим данные в табличной форме.



Из таблицы видно, что на табло обоих часов будет 6:00 в конце 15-го дня. Конечно, одни из них будут показывать 6:00 a.m., а другие — 6:00 p.m. Тем не менее показания совпадут, а именно это и требовалось найти в задаче.

Задача 7.10

Сколько существует положительных трехзначных нечетных чисел, произведение цифр которых дает число 252?

Обычный подход

Чаще всего начинают искать тройки множителей, произведение которых равно 252. Иначе говоря, выписывают наборы из трех цифр, дающие при перемножении 252. Это следует делать упорядоченно, начиная с цифр 1, 1, 252, за которыми следуют 1, 2, 126, затем 1, 3, 84, за ними 1, 4, 63 и т. д. Можно перебирать цифры таким образом до тех пор, пока не попадется хотя бы один набор, который даст нечетное трехзначное число. Не исключено, однако, что таких наборов несколько. Можно ли с уверенностью сказать, что найдены все возможные варианты? «Лобовой» подход не дает нам такой уверенности.

Образцовое решение

Попробуем применить нашу стратегию организации данных. Можно разложить число 252 на множители 2 x 2 x 3 x 3 x 7. Если одна из цифр 7, то другие цифры должны давать при перемножении 36, т. е. 4 и 9 или 6 и 6, поскольку все другие комбинации включают в себя множители, имеющие более одного знака. Комбинируя эти цифры с 7, мы находим пять чисел, которые удовлетворяют условиям задачи. Это 749, 479, 947, 497, 667 — все они нечетные трехзначные числа, а произведение их цифр равно 252.

Задача 7.11

Какое число в следующем ряду самое большое и какое число стоит на втором месте по величине?


Обычный подход

В наши дни рука автоматически тянется к калькулятору. Однако найти ответ может быть не так просто, как кажется, поскольку не все калькуляторы могут извлекать такие корни.

Образцовое решение

Сначала для удобства представим члены заданного ряда в виде степеней с дробными показателями:



Эффективной стратегией здесь является организация данных так, чтобы было легко сравнивать.


Задача 7.12

У класса есть возможность принять участие в походе. Желающих оказалось пять человек, а свободных мест — всего три. В числе желающих — Аманда, Билл, Кэрол, Дэн и Эван. Учитель взял пять листочков бумаги, каждый с именем одного из желающих, положил их в шляпу и вытащил три листочка наугад. Какова вероятность того, что Аманда, Билл и Кэрол попадут в число участников похода?

Обычный подход

Сначала определим, сколько вариантов выбора может существовать. Порядок не имеет значения, поэтому это комбинаторная задача. Из пяти листочков выбирают по три за раз:



Поскольку Аманда, Билл и Кэрол могут выпасть только один раз, ответ

Образцовое решение

Если вы не знакомы с комбинаторными вычислениями, то можете воспользоваться стратегией организации данных. Составим перечень всех возможных сочетаний имен в любом порядке:



Существует 10 возможных вариантов выбора трех человек. Только один из них, а именно ABC, удовлетворяет условиям задачи. Таким образом, правильным ответом будет один из 10, или

Глава 8
Схематичное изображение, или Визуальное представление

Когда вопросы в задаче касаются определенной геометрической фигуры или рисунка, без слов понятно, что построение схем, или визуальное представление, является неотъемлемой частью метода решения. Это необходимо и помогает решить задачу. Довольно трудно представить, чтобы математики в далекие времена оперировали геометрическими понятиями без рисунков или хотя бы демонстрировали свои геометрические расчеты без использования чертежа. Вместе с тем есть множество задач, где условия не предполагают построения чертежа, однако визуализация того, о чем идет речь, намного облегчает поиск решения. Многие люди лучше воспринимают информацию визуально — чтобы понять происходящее, им нужна картина, а не просто слова. Это не домыслы. Визуализация — очень сильный метод, помогающий вникнуть в данную ситуацию.

Например, когда нужно объяснить, как найти чей-то дом, схема направления движения просто неоценима. Рисунок помогает увидеть маршрут в целом. В журналах и газетах постоянно используются графики и другие визуальные инструменты для сравнения или противопоставления ситуаций. Когда вы покупаете что-то и должны собрать это сами, в руководстве производителя помимо письменных инструкций обычно приводятся рисунки. В большинстве видов спорта, особенно в футболе и баскетболе, тренер объясняет стратегию игры, как правило, с помощью диаграмм, или рисунков с крестиками и ноликами. Все это примеры повседневного использования стратегии схематичного изображения, когда напрямую оно не требуется. В конце концов, не зря же говорят, что лучше один раз увидеть, чем 100 раз услышать.

Возьмем для примера математическую задачу, в которой изначально мало кто ожидает использования визуального представления.

У г-на Адамса есть два теста в запасе для выпускного экзамена по алгебре, которые он хочет использовать в двух классах. В каждом тесте 26 разных вопросов. Он берет первые четыре вопроса из теста 1 и добавляет их в конец теста 2. Затем он берет четыре первых вопроса из теста 2 и добавляет их в конец теста 1. В каждом тесте теперь 30 вопросов. Сколько одинаковых вопросов в обоих тестах?

Ситуацию до и после перестановок можно представить схематично, или визуализировать ее:



Теперь видно, что тесты содержат восемь одинаковых вопросов, а именно 1, 2, 3, 4 и A, B, C, D. Хотя в этом случае не обязательно использовать визуальное представление, и задачу можно решить другими методами, создание схемы позволяет увидеть, что происходит. Такой подход облегчает поиск решения. Имейте в виду, когда мы говорим о визуальном представлении, не обязательно подразумевается «вычерчивание» чего-либо.

Вот еще одна задача, где визуальное представление помогает увидеть происходящее.

Длина стороны равностороннего треугольника равна 40 см. Средние точки сторон соединяются так, что образуется второй равносторонний треугольник. Средние точки сторон этого треугольника соединяются так, что образуется третий треугольник. Этот процесс продолжается до тех пор, пока мы не получим пять треугольников. Чему равен периметр пятого треугольника?

Излишне говорить, что в случае решения геометрической задачи — даже такой, которую легко изложить на словах, — построение чертежа очень полезно, а то и просто необходимо. Нам нужно видеть, о чем идет речь (рис. 8.1).

Чертеж должен показывать, что отрезок, соединяющий средние точки двух сторон треугольника, равен половине длины третьей стороны и параллелен ей. Таким образом, каждая сторона любого нашего треугольника равна длины соответствующей стороны предыдущего треугольника. Периметр каждого последующего треугольника равен половине периметра предыдущего треугольника. Для полной ясности составим таблицу, отражающую процесс.




Периметр пятого треугольника равен 7,5 см. Сделанный нами чертеж помог визуализировать ситуацию и решить задачу. Хотя решить ее можно и без рисунка, глядя на чертеж, легче найти ответ.

Чтобы подчеркнуть ценность использования схематичного представления, когда это напрямую не требуется в условиях, рассмотрим такую задачу.

В 5:00 часы бьют пять раз в течение 5 секунд. Сколько времени потребуется этим часам, чтобы пробить 10 раз в 10:00? (Предполагается, что на сами удары часов время не требуется.)

Ответ 10 секунд неверен! Характер этой задачи не предполагает создание каких-либо рисунков. Тем не менее представим ситуацию схематично, чтобы увидеть суть происходящего. На схеме каждая точка представляет удар часов. На рис. 8.2 мы видим, что общее время боя составляет 5 секунд, а между ударами четыре интервала.



Из этого следует, что интервал должен составлять секунды. Теперь рассмотрим второй случай на рис. 8.3.



Здесь мы видим, что между 10 ударами девять интервалов. Поскольку интервал равен секунды, продолжительность боя часов в 10:00 составит или секунды.

Схема значительно упростила задачу, которая иначе могла сбить человека с толку.

Стратегия составления схем или диаграмм, когда они не требуются по условиям задачи, нередко помогает найти решение, а в некоторых случаях прямо дает ответ, особенно в несложных задачах, где визуальное представление делает решение очевидным.

Задача 8.1

В классе г-на Страусса 25 столов, расставленных в виде квадрата в пять рядов по пять столов в каждом. Г-н Страусс хочет, чтобы все поменялись местами по такому правилу: каждый переходит за стол, находящийся слева или справа от него, или за стол, находящийся перед ним или позади него. Можно ли осуществить это?

Обычный подход

Самый распространенный подход к решению этой задачи — взять 25 листочков бумаги, которые будут обозначать столы, и подвигать их в соответствии с правилом г-на Страусса. Это неудобно, в процессе трудно уследить за всеми перемещениями и получить правильный ответ.

Образцовое решение

Вместо того, чтобы двигать листочки, давайте сделаем схему, или визуально представим расположение столов. Изобразим класс с 25 столами в виде шахматной доски, как показано на рис. 8.4.



Чтобы ученики соблюдали правило г-на Страусса, им нужно переходить с закрашенного стола на незакрашенный или наоборот. Однако у нас 13 закрашенных столов и только 12 незакрашенных. Таким образом, пересаживание по правилу г-на Страусса невозможно.

Задача 8.2

Стоимость разрезания и сваривания звена цепи составляет $1,00. У женщины есть семь звеньев, и она хочет сделать из них цепь. Во сколько долларов как минимум это обойдется?

Обычный подход

Наиболее очевидный подход — разрезать шесть звеньев, соединить их и сварить. Это будет стоить $6,00. Однако наверняка должен быть способ уменьшить затраты.

Образцовое решение

Воспользуемся стратегией визуального представления. Разомкнем звено 2 и соединим звенья 1, 2 и 3, как показано на рис. 8.5.



Разомкнем звено 5 и соединим звенья 4, 5 и 6, как показано на рис. 8.6.




Наконец, разомкнем звено 7 и соединим цепочки 1–2–3 и 4–5–6, как показано на рис. 8.7.

Поскольку нам пришлось разрезать только три звена, создание цепи обойдется в $3,00.

Задача 8.3

Если в среднем полторы несушки могут снести полтора яйца за полтора дня, то сколько яиц снесут шесть несушек за восемь дней?

Обычный подход

Это старая задача, которая выдержала испытание временем. Традиционно ее решают следующим образом. Поскольку несушки работают дня, можно сказать, что работа по откладыванию полутора яиц занимает или «несушко-дня». Аналогичным образом вторая работа занимает 6 x 8, или 48 «несушко-дней». Таким образом, можно составить пропорцию.

Обозначим за x количество яиц, снесенных 6 несушками за 8 дней. Тогда,



Перемножив средние и крайние члены пропорции, мы получим:


Образцовое решение

В качестве альтернативного подхода можно использовать следующее визуальное представление ситуации (в данном случае табличное):




Таким образом, 6 несушек должны снести 32 яйца за 8 дней.

Задача 8.4

Джек и Сэм заняты неполный рабочий день в местной пиццерии. Пиццерия открыта 7 дней в неделю. Джек работает день через два, а Сэм — день через три. Они работали вместе во вторник 1 марта. В какие еще дни марта Джек и Сэм будут работать вместе?

Обычный подход

Обычно берут два листа бумаги и выписывают на них раздельно все даты, в которые Джек и Сэм выходили на работу. Затем даты сравнивают и определяют совпадения. Это совершенно законный метод решения, который дает правильный ответ.

Образцовое решение

Более рациональный подход к этой задаче предполагает анализ с использованием визуального представления. Нарисуем календарь и просто поставим инициалы молодых людей в клетках с теми датами, в которые они работают.



В те даты, где стоят инициалы обоих молодых людей, они работают вместе. На рисунке эти даты ясно видны: 13 и 25 марта.

Эту задачу можно решить еще одним способом, если посмотреть на нее с другой точки зрения. Известно, что числа 4 и 3 являются взаимно простыми и представляют количество дней в рабочем цикле каждого молодого человека, соответственно. Их общее кратное, 12, дает дни между датами, в которые они работают вместе. Таким образом, 1 + 12 = 13 — это день, когда молодые люди работают вместе после первого дня, а 13 + 12 = 25 — это день, в который они работают вместе в следующий раз.

Задача 8.5

На местной ярмарке несколько работников занимаются отслеживанием количества людей, принимающих участие в конкретных мероприятиях каждый день. Записи Розалинды показывают, что с понедельника до субботы включительно стенд для стрельбы из лука посетили 510 человек. По подсчетам Габиэля с понедельника по среду включительно на этом стенде побывали 392 человека, а Фрэнк насчитал там во вторник и в пятницу 220 человек. Адель работала в среду, четверг и субботу и у нее получилось в сумме 208 человек. Наконец, в записях Альфреда значилось, что с четверга по субботу включительно на стенде побывали 118 человек. Если предположить, что все эти данные правильны, то сколько человек посетили стенд для стрельбы из лука в понедельник?

Обычный подход

Как правило, начинают составлять ряды уравнений, в которых переменные представляют разные дни недели. В результате получается пять уравнений первой степени с шестью неизвестными. Конечно, не все неизвестные встречаются в каждом уравнении.



Решив эту систему уравнений, можно попытаться найти ответ. Однако этот процесс довольно сложен и большинству не под силу. (Мало кто догадывается, что в результате вычитания уравнений 8.3 и 8.4 из уравнения 8.1 получается Пн. = 82.)

Образцовое решение

Визуализируем условия задачи в виде таблицы посещаемости стенда:



Обратите внимание на то, что за исключением понедельника каждый день упоминается три раза. Это приводит к двойному учету посетителей четырьмя последними учетчиками во все дни кроме понедельника. Таким образом, мы получаем одно уравнение:

2 x 510 - (392 + 220 + 208 + 118) = количество посетителей в понедельник; 1020 - 938 = 82.

В понедельник стенд посетили 82 человека.

Задача 8.6

Аманда, Айан, Сара и Эмили выставили своих лягушек для участия в соревнованиях на дальность прыжка на ярмарке. Лягушка Аманды опередила лягушку Эмили, но оказалась не первой. Лягушка Сары проиграла лягушке Аманды, но была не последней. Как распределились места лягушек?

Обычный подход

Чаще всего берут четыре фишки, жетона или монеты, наклеивают на них стикер с именем владельца и переставляют этих «лягушек» до тех пор, пока результат не будет удовлетворять условиям задачи.

Образцовое решение

Эту задачу проще решить с использованием визуального представления. Прежде всего, мы знаем, что лягушка Аманды опередила лягушку Эмили, но была не первой. Обозначим это схематично так:



Лягушка Сары проиграла лягушке Аманды, но была не последней. Продолжив построение схемы, мы получаем следующее распределение мест:



Схема позволила легко увидеть порядок, в котором распределились места.

Задача 8.7

Из 40 мальчиков в оздоровительном лагере «Кэмп-Уолден» 14 участвовали в заплыве на озере, 13 играли в баскетбол, а 16 ходили в поход. Трое мальчиков играли в баскетбол и участвовали в заплыве. Пять мальчиков участвовали в заплыве и ходили в поход. Восьмеро мальчиков играли в баскетбол и ходили в поход, а двое мальчиков участвовали во всех трех спортивных мероприятиях. Сколько мальчиков в этом лагере не участвовали ни в чем?

Обычный подход

Традиционно эту задачу начинают решать путем сложения всех участников спортивных мероприятий с последующим вычитанием повторов. Такая процедура редко бывает успешной.

Образцовое решение

Попробуем применить для решения задачи визуальное представление. Для наглядного отображения данных используем диаграмму Венна (рис. 8.8).



Область наложения всех трех кругов представляет двоих мальчиков, которые участвовали во всех трех спортивных мероприятиях. Круги показывают следующее:

Участвовали в заплыве = 14;

Играли в баскетбол и ходили в поход = 8;

Участвовали в заплыве и играли в баскетбол = 3;

Играли в баскетбол = 13;

Участвовали в заплыве и ходили в поход = 5;

Ходили в поход = 16.

При сложении этих частей диаграммы Венна мы получаем 8 + 3 + 2 + 1 + 4 + 6 + 5 = 29. В лагере было 40 мальчиков, из которых 29 участвовали в спортивных мероприятиях, а 11 нет.

Задача 8.8

Сколько целых чисел, цифры которых расположены в порядке возрастания, находится между 4000 и 5000?

Обычный подход

К решению этой задачи можно подойти, сообразив, что первой цифрой должна быть 4, а значит, на втором месте может стоять цифра 5, 6 или 7. Цифры 8 и 9 для этого не подходят, поскольку вслед за ними в возрастающем порядке уже ничего не расположишь. В результате таких рассуждений должно получиться следующее: 4567, 4568, 4569, 4578, 4579, 4589, 4678, 4679, 4689 и 4789.

Образцовое решение

Чтобы подойти к решению более организованно, воспользуемся схемой, представленной на рис. 8.9, хотя задача по своему характеру не требует никаких рисунков.

Каждый путь, начинающийся от цифры 4, ведет к числу, которое находится в диапазоне между 4000 и 5000. Всего таких путей 10, и они дают следующие числа: 4567, 4568, 4569, 4578, 4579, 4589, 4678, 4679, 4689 и 4789. Таким образом, мы получаем искомые числа с помощью схемы, построения которой условия задачи не требуют.


Задача 8.9

У моего брата целая коллекция фигурок двуногих обезьян и четвероногих буйволов. Если в коллекции всего 100 фигурок и в сумме 260 ног, то сколько в ней фигурок каждого вида?

Обычный подход

Чаще всего составляют два уравнения и решают их. Обозначим число фигурок обезьян как a, а число фигурок буйволов как b. Тогда мы получаем следующие уравнения:

a + b = 100;

2a + 4b = 260.

Умножение первого уравнения на 2 дает:

2a + 2b = 200;

2a + 4b = 260.

Если вычесть первое уравнение из второго, то мы получим:

2b = 60;

b = 30.

Таким образом, в коллекции 30 буйволов и 70 обезьян.

Образцовое решение

Воспользуемся визуальным представлением данных (нарисуем схему), чтобы решить задачу. Прежде всего, уменьшим числа в условиях задачи в 10 раз, чтобы ими было легче оперировать (но будем помнить о том, что полученный результат нужно умножить на 10 для восстановления исходного порядка чисел). Итак, теперь у нас всего 26 ног и 10 фигурок. Нарисуем 10 окружностей, которые будут представлять 10 фигурок. Независимо от того, что это за фигурка, обезьяна или буйвол, у нее должно быть не менее двух ног (рис. 8.10).

До нужной величины нам не хватает шести ног — их необходимо добавлять парами (рис. 8.11).

У нас получилось три четвероногих фигурки и семь двуногих. Осталось умножить их на 10. Таким образом, мы получаем 30 фигурок буйволов и 70 фигурок обезьян.



Глава 9
Учет всех возможностей

Мы знаем, что организация данных иногда очень облегчает поиск решения. Если нужно выявить, например, закономерность, то аккуратное представление данных в виде списка или таблицы может помочь в этом. Особенно интересны здесь исчерпывающие списки, т. е. списки, в которых систематизированно перечисляются все существующие возможности. В таких списках часто обнаруживается то, что мы ищем. Составление исчерпывающего списка позволяет тщательно проанализировать все возможности.

В качестве примера предположим, что у вас не работает лампа. Попробуем перечислить все возможности. (Конечно, это можно сделать мысленно, но в результате вы все равно получите список.) Проблема может крыться в перегоревшей лампочке, оборванном проводе, неработающей розетке, сработавшем предохранителе или неисправном выключателе. Проверяя возможности одну за другой, мы в конечном итоге дойдем до той, которая является причиной неисправности. Математический пример может выглядеть так:

Имеется двузначный квадрат целого числа. Если вставить одну цифру между существующими двумя, то получится трехзначный квадрат целого числа. Какие трехзначные квадраты чисел мы получаем?

Проанализируем все возможности. Прежде всего, составим исчерпывающий список двузначных квадратов целых чисел, их шесть:

16, 25, 36, 49, 64, 81.

Теперь составим исчерпывающий список трехзначных квадратов целых чисел:

100, 121, 144, 169, 196, 225, 256, 289, 324, 361, 400, 441, 484, 529, 576, 625, 676, 729, 784, 841, 900, 961.

Выберем из второго списка те числа, которые можно составить, вставив какую-либо цифру между первой и второй цифрами двузначных квадратов целых чисел. Такому условию удовлетворяют только 196 (вставлена 9 между цифрами числа 16), 225 (вставлена 2 между цифрами числа 25) и 841 (вставлена 4 между цифрами числа 81). Два исчерпывающих списка сделали очевидными все возможности. Обратите внимание на то, что исчерпывающий список не только содержит ответ задачи, но ограничивает количество исследуемых возможностей.

Вот еще один пример использования этой полезной стратегии.

На скамейке в парке сидят два человека. Один из них — женщина. Какова вероятность того, что и второй тоже окажется женщиной?

Составим список всех возможностей (М = мужчина, Ж = женщина):

М — М М — Ж Ж — М Ж — Ж.

В список пошли четыре возможности, однако в нашей задаче первую, М — М, не нужно учитывать, поскольку известно, что как минимум один человек — женщина. У нас остаются три варианта, и лишь в одном из них могут быть две женщины. Таким образом, ответом на поставленный вопрос будет вероятность, равная.

Чтобы еще лучше увидеть ценность такого подхода к решению задач, рассмотрим еще один пример:

В двух залах местного кинотеатра показывают по утрам разные мультфильмы. Утренние сеансы в обоих залах должны заканчиваться к 13:00, когда начинается демонстрация художественных фильмов. В зале A первый сеанс мультфильмов начинается в 9:00, второй в 9:28, а потом через каждые 28 минут. В зале B первый сеанс тоже начинается в 9:00, но потом сеансы повторяются через 35 минут. Джоанн хочет попасть на просмотр мультфильмов в обоих залах. Во сколько два последующих сеанса начинаются одновременно?

Составим исчерпывающий список времени начала сеансов в обоих залах.



Любой последующий сеанс должен был бы начаться уже после 13:00. Мы перечислили все возможности! Где-то в этом списке всех должен находиться ответ. Список показывает лишь одно время, когда начало сеансов в обоих залах совпадает — 11:20.

Такая стратегия очень эффективна, но вы должны убедиться в том, что перечислили все без исключения возможности! Только тщательная организация данных может дать вам полную уверенность. Как и в случае с другими стратегиями, необходимо обдуманно подходить к выбору той, которая подходит в данном случае. Стратегия учета всех возможностей может сделать решение более очевидным.

Задача 9.1

Учитель математики замечает, что его нынешний возраст представляет собой простое число. Он обнаруживает, что в следующий раз его возраст станет простым числом через столько же лет, сколько прошло с той поры, когда возраст был простым числом в прошлый раз. Сколько лет учителю математики?

Обычный подход

У этой задачи не так много альтернативных способов решения. Обычно начинают перебирать числа в надежде «наткнуться на подходящее».

Образцовое решение

Здесь наверняка нам пригодится стратегия учета всех возможностей. Рассмотрим следующий список:



В списке простых чисел от 1 до 100 (хотя в ситуации учителя математики можно было бы ограничиться числами в диапазоне от 20 до 80) только в двух случаях три последовательных простых числа имеют одинаковую разность. Первый случай — 3, 5 и 7 — нам не подходит, поскольку пятилетних учителей математики не бывает. Второй случай — 47, 53 и 59 — укладывается в подходящий возрастной диапазон. Таким образом, учителю математики должно быть 53 года.

Задача 9.2

Найдите количество сочетаний, при которых 20 монет достоинством 5 центов, 10 центов и 25 центов могут составить в сумме $3,10.

Обычный подход

Большинство людей сразу начинают составлять алгебраические уравнения, отражающие информацию из условий задачи. В результате они получают: n + d + q = 20, где n, d и q — количество 5-, 10- и 25-центовых монет соответственно. Это можно записать, как n = 20 — q — d. Кроме того, 25q + 10d + 5n = 310, что при объединении с предыдущими двумя уравнениями дает: 25q + 10d + 5 (20 — q — d) = 310. Отсюда 4q + d = 42, или После этого остается лишь подставлять разные значения, чтобы выявить наилучший результат.

Образцовое решение

У нас, однако, есть более рациональный метод, а именно учет всех возможных значений d. Прежде всего, мы замечаем, что q должно быть целым числом, а значит необходимо выделить дробную часть q, т. е. или d = 2–4k. Подставив это в приведенное выше уравнение, мы получаем q = 10 + k, а n = 20 — q — d = 20 — (10 + k) — (2–4k), или n = 8 + 3k.

Поскольку d = 2–4k, значение k может быть либо нулевым, либо отрицательным.

В таблице ниже приведены возможные значения k и вытекающие из него значения d, q и n.



При k = 0, –1, –2 мы получаем реальные варианты. Когда же k = –3, d = 2–4(–3) = 14, а n = 8 + 3(–3) = –1, что не имеет смысла в этой задаче. Таким образом, количество сочетаний, при которых сумма составляет $3,10, равно трем.

Задача 9.3

Для доставки консервов из тунца компания может использовать небольшие коробки, в которые входит восемь банок, и коробки побольше, вмещающие 10 банок. С целью экономии компания старается чаще использовать большие коробки. Если заказ составляет 96 банок, то как лучше упаковать его для отправки?

Обычный подход

Эта задача имеет любопытное математическое решение. Если обозначить как x количество небольших коробок, а как y количество больших коробок, то мы получим уравнение:

8x + 10y = 96.

В этом уравнении, однако, две неизвестные, что обычно означает наличие множества решений. Такое уравнение, где значения x и y могут быть только целыми, называют диофантовым по имени древнегреческого математика Диофанта (примерно 208–292 гг. н. э.). Попробуем решить его. Выразим x через y:



Значение члена должно быть целым, чтобы получить целое значение x. Пусть y = 4. Тогда а x = 12 - 4–1 = 7. Таким образом, мы получаем семь маленьких коробок и четыре больших. Могут ли существовать другие ответы? Попробуем поискать их. Аналогичным образом можно принять y = 0 и получить 12 и 0. Наконец при y = 8 мы получаем x = 2.

Образцовое решение

Для решения этой задачи лучше всего подходит стратегия учета всех возможностей и представление данных в табличной форме.



Похоже, мы сразу получили один правильный ответ! Он удовлетворяет численным условиям задачи — необходимо отправить 96 банок. Вместе с тем единственная ли это возможность? В конце концов, такой ответ означает, что компания не использует ни одной большой коробки. А из условий нам известно, что она стремится использовать максимальное количество больших коробок. Поэтому продолжим таблицу и попробуем найти все возможные варианты.



Существуют три варианта упаковки: 2 маленьких коробки и 8 больших; 7 маленьких коробок и 4 больших; 12 маленьких коробок и 0 больших. Вместе с тем, поскольку компания стремится использовать максимальное количество больших коробок, ответ для нашей задачи — 2 маленькие коробки и 8 больших. Обратите внимание на то, что с математической точки зрения все три ответа удовлетворяют условию, в соответствии с которым необходимо отправить 96 банок. Контекст задачи, однако, заставляет отбросить два варианта ответа из тех трех, что позволила выявить таблица.

Задача 9.4

На стандартном игральном кубике точки на противоположных гранях составляют в сумме 7. Сколько разных сумм дают точки на трех соседних гранях стандартного кубика?

Обычный подход

Одни обычно пытаются нарисовать кубик и последовательно подсчитать точки на соседних гранях. Другие начинают выписывать возможные сочетания точек на трех гранях, независимо от того, находятся они рядом или нет.

Образцовое решение

Организуем данные таким образом, чтобы можно было учесть все возможности. Поскольку сумма точек на противоположных гранях равна 7, варианты сочетания могут быть лишь такими:

1 и 6;

2 и 5;

3 и 4.

Известно, что у трех соседних граней должна быть общая вершина. Всего вершин у кубика восемь, поэтому наборов из трех соседних граней тоже должно быть восемь. Посмотрим, разные у них суммы точек или нет. Для этого перечислим все возможные сочетания по три, выбирая одно число на описанных выше трех парах противоположных граней, а потом определим их суммы. Чтобы не пропустить ни одной возможности, выбор будем проводить упорядоченно:



Существует восемь разных сумм, как и следовало ожидать при восьми вершинах.

Задача 9.5

Во время последней переписи населения респондент сказал переписчику, что у него трое детей. Когда его спросили об их возрасте, он ответил, что не может сказать этого, но произведение их возрастов равно 72, а сумма такая же, как номер нашего дома. Переписчик выбежал на улицу, посмотрел номер дома и сказал, что все равно не понимает. Тогда респондент добавил: «Ах да, я забыл сказать, что мой старшенький любит блинчики с черникой». После этого переписчик быстро заполнил графу возраста. Сколько лет детям респондента? (Рассматривайте только целые числа.)

Обычный подход

Самый распространенный подход — попытаться составить ряд уравнений. Если обозначить возраст трех детей, как x, y и z, то мы получаем:



Здесь мы заходим в тупик: у нас система из двух уравнений с четырьмя неизвестными. Такую задачу, похоже, невозможно решить. Можно, конечно, попробовать угадать, но на это потребуется уйма времени.

Образцовое решение

Воспользуемся стратегией учета всех возможностей. Поскольку произведение возрастов равно 72, начнем с перечисления всех троек чисел, которые при перемножении дают 72. Не будем забывать также об аккуратной организации данных, чтобы не упустить какую-нибудь возможность.



Это полный список триад чисел, произведение которых равно 72. Где-то в нем прячется ответ. Мы также знаем, что сумма возрастов равна номеру дома:



Переписчик видел номер дома, однако все равно не мог определить возраст детей. Почему? Если, например, номер дома равен, скажем, 18, то возраст становится очевидным — 1, 8 и 9. Однако невозможность определить правильную тройку чисел предположительно связана с тем, что есть две тройки чисел, дающие в сумме 14. Иначе говоря, у дома должен быть номер 14. Вместе с тем, как только респондент произнес слова «мой старшенький любит блинчики с черникой», переписчик понял, что один ребенок должен быть старшим. В этом случае возраст детей равен 3, 3 и 8, поскольку в тройке 2, 6 и 6 нет единственного старшего.

Обратите внимание на то, что блинчики с черникой на деле всего лишь отвлекающие слова. Ключом к решению задачи является слово «старшенький».

Задача 9.6

Сколько общих для двух окружностей касательных можно провести на рис. 9.1?


Обычный подход

Можно попробовать построить общие касательные и сосчитать их, однако нет никакой гарантии, что это будут все касательные, поскольку рисунок довольно обманчив.

Образцовое решение

Чтобы организованно подойти к решению этой задачи, нужно брать по две окружности за раз и учитывать все возможности.

Окружности A и B: 2 внешних касательных + 1 внутренняя;

Окружности A и C: 2 внешних касательных + 2 внутренних;

Окружности B и C: 2 внешних касательных.

Таким образом, суммарное количество общих касательных равно девяти. Задача легко решается путем учета всех возможностей.

Задача 9.7

Мария помогает отцу укладывать плитку на пол прямоугольной комнаты для игр. Всего у них ушло ровно 2005 квадратных плиток двух цветов — черного и белого. Периметр в одну плитку шириной был полностью черным. Остальная плитка имела белый цвет. Сколько белых плиток потребовалось, чтобы покрыть пол?

Обычный подход

Если сделать рисунок, то мы получим два прямоугольника, как показано на рис. 9.2. Если размеры внутреннего прямоугольника x и y, то ширина внешнего равна x + 2, а длина — y + 2.



Естественная реакция — представить полученную информацию алгебраически в форме уравнения:

(x + 2) (y + 2) = 2005.

Выполнив умножение и упростив это уравнение, мы получим:

xy + 2y + 2x + 4 = 2005;

xy + 2y + 2x = 2001.

Мы получили одно уравнение с двумя неизвестными, и нам нужно найти xy. Это тупик, а не решение.

Образцовое решение

Подойдем к имеющейся информации с другой стороны и рассмотрим все возможности. Количество плиток, 2005, можно разложить на множители только двумя путями: либо 1 x 2005, либо 5 x 401. Это дает два возможных размера искомого прямоугольника. Первую ситуацию можно отбросить, поскольку при ширине в одну плитку для белых плиток места нет. Следовательно, в игровой комнате должно быть 5 x 401 плиток. Поскольку снаружи выполнена «рамка» шириной в одну плитку, размеры внутреннего прямоугольника из белых плиток на две плитки меньше в каждом направлении. Если уменьшить каждое направление на две плитки, то количество белых плиток для внутреннего прямоугольника составит 3 x 399, или 1197. Таким образом, для покрытия пола было использовано 1197 белых плиток.

Задача 9.8

Даны целые числа от –100 до +100. Сколько таких чисел при возведении в квадрат имеют цифру 1 в разряде единиц?

Обычный подход

Естественная реакция — начать с выписывания всех целых чисел от 1 до 100. Затем их по очереди возводят в квадрат и подсчитывают те, у которых в конце стоит 1. Результат после этого удваивают, чтобы учесть числа от –1 до –100.

Образцовое решение

Воспользуемся стратегией учета всех возможностей. Единственными числами, квадраты которых могут иметь цифру 1 в разряде единиц, являются те, что оканчиваются на 1 или 9. Таким образом, существует всего 20 возможностей, а именно 1, 11, 21, 31, 41, 51, 61, 71, 81, 91, 9, 19, 29, 39, 49, 59, 69, 79, 89 и 99. Удвоив это количество, чтобы учесть все возможности в отрицательном диапазоне, мы получаем ответ — 40 целых чисел.

Задача 9.9

На рис. 9.3 показаны три грани куба. Если продолжить нумерацию на остальных гранях куба, то чему будет равна сумма номеров всех шести граней?


Обычный подход

Большинство людей замечают, что числа на гранях куба начинаются с 48 и 49. Чаще всего они просто продолжают числовой ряд и получают следующие номера на гранях: 48, 49, 50, 51, 52, 53. Поскольку номер третьей грани, а именно 52, присутствует в этой последовательности, некоторые останавливаются и дают в качестве ответа сумму перечисленных номеров — 303.

Образцовое решение

Вместе с тем в приведенном выше решении учтены не все возможности. Мы видим три грани из шести. Поскольку нам видны номера 48, 49 и 52, значит обязательно должны быть 50 и 51. Шестой номер, однако, может находиться на любом конце последовательности. Таким образом, существуют два варианта шестого номера — 47 или 53. Это дает две возможные суммы — 297 и 303.

Глава 10
Обоснованное предположение и проверка

Конечно, даже мысль о том, что в качестве стратегии решения задач можно использовать догадки, вызывает недоумение. В самом деле, может ли кто из нас вспомнить, чтобы учитель говорил кому-то, давшему нестандартный ответ: «Ты это знаешь или просто строишь догадки?» В некоторых книгах выдвижение предположений и их проверку называют методом «проб и ошибок», и это воспринимается более негативно. Добавление определения обоснованное в название метода должно успокоить вас и уверить в том, что это действительно эффективная и нередко очень полезная стратегия.

Мы пользуемся стратегией «предположение-и-проверка» на протяжении всей своей жизни. Например, на кухне мы делаем предположение о том, готово ли мясо в духовке, а потом с помощью специального термометра проверяем, правильно ли оно. Если нет, то мясо опять отправляется в духовку, а процесс «предположение-проверка» повторяется через некоторое время. Пытаясь найти конкретное место во время поездки на автомобиле, мы «предполагаем», что оно находится на определенной улице. Если его там нет, то мы делаем другое предположение на основе информации, полученной в результате первой попытки.

При решении задач, когда слишком много неопределенностей, мы можем использовать эту стратегию для уменьшения неопределенности с помощью конкретных предположений. Проверяя предположения, мы получаем информацию для уточнения следующего предположения и приближаемся к отысканию ответа.

Чтобы помогать решению задач, предположения должны быть неслучайными и не взятыми с потолка без очевидного основания. После изучения условий задачи определяется возможный подход к решению и выдвигается предположение. Затем предположение проверяется на основе условий задачи. Если это не приводит к получению ответа, то выдвигается следующее предположение с учетом информации, полученной на предыдущем этапе. Новое предположение опять проверяется. Процесс уточнения предположений продолжается до тех пор, пока не будет накоплена информация, достаточная для решения задачи. Например, предположим, что нас просят найти следующие два члена последовательности 2, 0, 4, 3, 6, 7, 8, 12, 10, 18, ____, ____. Что мы видим? Возможно, члены последовательности возрастают и уменьшаются случайным образом. Не исключено, что здесь смешаны две последовательности. Это выглядит как обоснованное предположение. Попробуем проверить его!



Похоже, что наше предположение правильно — здесь действительно две последовательности. Последовательность 1 состоит из четных чисел. Следующим ее членом будет 12. В последовательности 2 разность между последовательными членами возрастает каждый раз на 1, т. е. разности равны 3, 4, 5, 6 и т. д. Следующий ее член должен быть равен 25. Таким образом, мы получаем ответ: следующие два члена — это 12 и 25. Обратите внимание на то, что здесь использовалась также стратегия выявления закономерности. В применении нескольких стратегий для решения задачи нет ничего необычного.

Обратите внимание также, что предположения берутся не с потолка. Все они основываются на тщательном анализе того, что дано, и того, что требуется найти. Предположения делаются с умом! Не забывайте, что эту стратегию не случайно называют «обоснованным предположением и проверкой».

Рассмотрим еще один пример использования этой стратегии.

Местная фирма должна выполнить заказ на поставку полых и сплошных резиновых шаров. Полый шар весит одну унцию, а сплошной — две унции. И те и другие шары имеют одинаковый размер. В коробку вмещается 50 шаров. Самый выгодный транспортный тариф установлен для коробок весом 80 унций. Сколько шаров того и другого вида нужно положить в коробку, чтобы получить этот вес?

Вместо привычного составления уравнений попробуем воспользоваться стратегией выдвижения обоснованного предположения и его проверки. Для отслеживания наших предположений составим таблицу. Начнем с середины — с 25 шаров каждого вида.



В коробку следует положить 30 сплошных шаров и 20 полых. Если попробовать все сочетания подряд, то они все равно приведут к правильному ответу. Выдвижение обоснованных предположений позволяет сократить количество попыток.

Рассмотрим еще одну задачу, решение которой сильно выигрывает от применения нашей стратегии.

Игра в дротики очень популярна во многих странах. Памела сделала несколько бросков в мишень, секции которой обозначены как 2, 3, 5, 11 и 13. Если ее счет составил 150, то какое наименьшее количество дротиков она могла бросить?

Поскольку нужно найти минимальное количество дротиков, секций с высокими значениями должно быть как можно больше. Сделаем несколько предположений и представим результаты в табличной форме.



Наименьшее количество дротиков, которые могли потребоваться Памеле, равно 12. Обратите внимание на то, что мы опять использовали стратегию организации данных для отслеживания результатов оценки предположений. Табличное представление данных нередко очень облегчает анализ полученной информации.

Задача 10.1

На местной ферме выращивают голубику, кусты которой высажены так, что они образуют решетку с квадратными ячейками, а количество рядов равно количеству колонок. Фермер решил увеличить размеры поля на одинаковое количество рядов и колонок. Новое поле вмещает на 211 кустов больше, чем старое. Сколько кустов было в одном ряду на старом поле?

Обычный подход

Обычно начинают с составления уравнений. Возьмем за x количество рядов и колонок. Тогда первоначальное количество кустов равно x x x, или x2. Обозначим дополнительные кусты в каждом ряду и колонке как b, тогда новое количество кустов будет равно (x + b)2. Теперь у нас есть уравнение:

x2 + 211 = (x + b)2;

x2 + 211 = x2 + 2bx + b2;

211 = b2 + 2bx.

Здесь возникает проблема. Мы получили квадратное уравнение с неизвестной b, в котором есть еще одна неизвестная x. Что с ним делать? Возможно стоит подставить какие-нибудь значения вместо неизвестных и посмотреть, не удастся ли решить уравнение. Хотя такой подход и может дать правильный ответ, он не слишком эффективен.

Образцовое решение

Попробуем пойти путем выдвижения предположений и их проверки. Мы видим, что 211 — это простое число, а x и b должны быть целыми числами. Если разложить на множители приведенное выше уравнение, то мы получим:

211 = b (b + 2x).

Поскольку 211 — это простое число, у него только два множителя: 211 и 1. Таким образом, b должно быть равным 1, а (b + 2x) — 211. В результате мы получаем 2x = 210, а x = 105. В одном ряду на старом поле было 105 кустов.

Задача 10.2

Джек хочет огородить прямоугольный участок, отведенный под огород. У него есть готовая ограда длиной 20 м. Какие размеры должен иметь участок, чтобы огороженная площадь была наибольшей?

Обычный подход

Наиболее очевиден алгебраический подход. Можно составить уравнения, а потом решить их. Обозначим длину, как x, а ширину, как y. Тогда мы получим:

2x + 2y = 20, или x + y = 10.

При составлении второго уравнения возникает проблема — как представить максимальную площадь? Иначе говоря, нам нужно получить xy = максимум. Что здесь можно сделать? Посмотрим, можно ли найти другой подход.

Образцовое решение

Первая же прикидка показывает, что, например, длина 8 и ширина 2 «подходят». Однако точно так же подходят и другие пары чисел. Воспользуемся стратегией обоснованного предположения и проверки, чтобы понять, какие размеры дают наибольшую площадь. Будем вести учет предположений в табличной форме. Поскольку для определения площади нужно умножить одну длину на одну ширину, ограничимся половиной периметра, равной 10. Начнем с наибольшей возможной длины.



Похоже, что прямоугольник размером 5 x 5 (квадрат) имеет наибольшую площадь. А что, если попробовать дробные размеры? В условиях задачи не говорится, что они должны быть целыми. Добавим в нашу таблицу дробные значения и посмотрим, что произойдет.



Все равно получается, что прямоугольник с периметром 20 м имеет наибольшую площадь при размерах 5 x 5 (квадрат). Некоторые и без этого знают, что при заданном периметре прямоугольника наибольшую площадь всегда имеет квадрат. А раз так, то ответ получается совсем быстро — это квадрат с периметром 20, площадь которого равна 5 x 5 = 25 м2.

Задача 10.3

Найдите наименьшее простое число, превышающее 510. (Напомним, что простым называют такое число, которое делится только на 1 и на само себя.)

Обычный подход

Поскольку в задаче требуется найти наименьшее простое число, превышающее 510, мы будем, начиная с 511, брать число и пробовать разные делители в порядке возрастания вплоть до его половины. Если ни один из этих возможных делителей не подойдет, значит мы нашли простое число.

Образцовое решение

Воспользуемся стратегией обоснованного предположения и проверки для сужения диапазона возможных вариантов. Мы знаем, что число, превышающее 510, не может быть простым, если у него в конце стоят цифры 0, 2, 4, 5, 6 или 8. Кроме этого вспомним, что число, сумма цифр которого делится на 3, тоже делится на 3. Это позволяет отбросить некоторые числа, превышающие 510, например число 513. Таким образом, мы ограничиваем предположения числами 511, 517, 521 и т. д. В результате проверки следующим за 510 простым числом оказывается 521.

Задача 10.4

В эстафетном забеге на одну милю участвует команда в составе: Густав, Йохан, Ричард и Вольфганг. Они бегут свой четвертьмильный этап в том порядке, в котором перечислены. Каждый бегун проходит свой этап на 2 секунды быстрее предыдущего. Они финишируют с общим временем 3 минуты 40 секунд. За сколько каждый бегун пробежал свой этап?

Обычный подход

Применив несложные алгебраические вычисления, можно решить задачу следующим образом:

x = время, за которое свой этап пробежал Густав;

x — 2 = время, за которое свой этап пробежал Йохан;

x — 4 = время, за которое свой этап пробежал Ричард;

x — 6 = время, за которое свой этап пробежал Вольфганг.

x + (x — 2) + (x — 4) + (x — 6) = 220

(3 минуты 40 секунд = 220 секунд);

4x — 12 = 220;

4x = 232;

x = 58.

Густав пробежал свой этап за 58 секунд, Йохан — за 56 секунд, Ричард — за 54 секунды, Вольфганг — за 52 секунды.

Образцовое решение

Конечно, это решение зависит от знания алгебраических методов. Вместе с тем задачу можно решить с помощью стратегии обоснованного предположения и проверки. Предположим, что бегуны прошли дистанцию примерно с одинаковой скоростью. Если так, то можно разделить 220 на 4 и получить 55 в качестве первого предположения.



Таким образом, Густав пробежал свой этап за 58 секунд, Йохан — за 56 секунд, Ричард — за 54 секунды, Вольфганг — за 52 секунды.

Задача 10.5

В коробке у Дэна находятся почтовые марки стоимостью 13 и 8 центов. Отправка посылки, которую он приготовил, стоит ровно $1. Сколько марок каждого достоинства Дэн должен наклеить на посылку?

Обычный подход

Можно попробовать решить эту задачу алгебраически. Если обозначить как x количество 13-центовых марок и как y количество 8-центовых марок, то мы получим следующее уравнение:

0,13x + 0,08y = 1,00.

Если перевести все в центы, то уравнение приобретет вид:

13x + 8y = 100.

Это, однако, уравнение с двумя неизвестными, а значит ответов может быть несколько. Поскольку количество марок должно быть целым числом, нам нужно решить диофантово уравнение.

Для начала выразим y через x: После деления и выделения целых величин и остаточных членов, а затем объединения остаточных членов мы получаем:

Дробная часть должна быть целым числом, поскольку количество марок не может быть дробным. Выберем какое-нибудь значение для x, при котором дробная часть превращается в целое число. Пусть x = 4. Тогда y = 12 - 4 + (–2), или y = 6.

Дэн, таким образом, должен использовать шесть 8-центовых марок и четыре 13-центовых марки. (Но все ли это возможности? Можно ли найти все возможные ответы?)

Образцовое решение

Более изящное решение дает использование нашей стратегии обоснованного предположения и проверки в сочетании с табличным представлением результатов.



Таким образом, четыре 13-центовых марок и шесть 8-центовых марок дают сумму $1, необходимую Дэну. Обратите внимание на то, что таблица ясно показывает отсутствие других вариантов.

Задача 10.6

Разница между двумя положительными целыми числами равна 5. Если сложить их квадратные корни, то сумма также будет равна 5. Что это за целые числа?

Обычный подход

Традиционный подход — это составление системы уравнений:

Пусть x = первое целое число;

Пусть y = второе целое число.

Тогда:



Возведем обе стороны в квадрат:



Упростим полученное выражение:



Снова возведем обе стороны в квадрат:

4x2 + 20x = 4x2 — 80x + 400;

100x = 400;

x = 4;

y = 9.

Два целых числа — 4 и 9.

Образцовое решение

Традиционный подход требует умения решать уравнения с радикалами и связан с большим количеством алгебраических преобразований. В качестве альтернативы воспользуемся нашей стратегией обоснованного предположения и проверки. Поскольку сумма квадратных корней из двух целых чисел равна 5, квадратные корни этих чисел должны представлять собой 4 и 1 или 3 и 2. Таким образом, целые числа должны быть равными 16 и 1 или 9 и 4. Вместе с тем, если взять разность, которая равна 5, становится понятно, что правильный ответ — 9 и 4.

Задача 10.7

Тренер футбольной команды разрешает игрокам самостоятельно выбрать номер, под которым они выйдут на поле. Макс и Сэм, которые не только играют в футбол, но и входят в состав математической команды, останавливаются на особой паре номеров. Когда их номера возводят в квадрат, они дают двузначные числа. Когда два футболиста стоят рядом, образующееся из этих квадратов четырехзначное число также является квадратом простого числа. Какие номера они выбрали?

Обычный подход

Большинство людей берут числа 1, 2, 3, 4, 5, … и возводят их в квадрат, пытаясь найти те, которые дают двузначный квадрат. Затем они помещают эти квадраты рядом друг с другом и смотрят, какие из них образуют квадрат простого числа. Такое гадание нельзя назвать продуктивным.

Образцовое решение

Призовем на помощь нашу стратегию обоснованного предположения и проверки. Прежде всего, можно ограничить количество чисел, из которых делается выбор. При возведении в квадрат двузначное число дают числа от 4 до 9, поскольку квадраты 1, 2 и 3 — это однозначные числа, а квадраты 10, 11, …, 31 — трехзначные числа. Таким образом, мы можем выбирать из следующих квадратов: 16, 25, 36, 49, 64, 81. Начиная с 16 проверим, пара каких квадратов образует при размещении рядом квадрат простого числа. Обратите внимание, если мы оцениваем 1625 (это не квадрат простого числа), то нам нужно оценить и 2516 (тоже не квадрат простого числа). Чтобы выдвинуть обоснованное предположение, нужно в пару к 16 поставить оставшиеся двузначные числа. Если взять пару 16 и 81, то мы получим число 1681, равное 412. Макс и Сэм выбрали в качестве своих номеров числа 4 и 9.

Обратите внимание на то, что числа 3 и 4 тоже работают, так как 32 = 9, а 42 = 16. При размещении рядом друг с другом эти квадраты дают число 169, которое является квадратом простого числа. Однако в условиях задачи говорится о четырехзначном числе, так что этот ответ исключается.

Задача 10.8

Лайза получила на неделю задание решить 26 арифметических задач. Чтобы заинтересовать ее, отец обещал выдавать ей по 8 центов за правильно решенные задачи и вычитать по 5 центов за неправильно решенные. После выполнения задания Лайза обнаружила, что отец не должен ей ничего, но и она ничего не должна. Сколько задач Лайза решила правильно?

Обычный подход

Эту задачу позволяет решить обычный алгебраический подход.

Пусть x обозначает количество правильно решенных задач, а y — количество неправильно решенных задач.

Тогда:

8x — 5y = 0;

x + y = 26.

Из первого уравнения получаем, что 8x = 5y и

Подстановка значения x во второе уравнение дает:



Лайза решила правильно 10 задач и неправильно 16 задач.

Образцовое решение

Те, кто не умеет решать системы из двух уравнений с двумя неизвестными, могут попробовать найти ответ с помощью стратегии обоснованного предположения и проверки. Результаты лучше представлять в табличной форме. Начнем с середины — 13 правильных решений и 13 неправильных.



Лайза решила правильно 10 задач и неправильно 16 задач.

Табличное представление результатов делает ответ очевидным. Обратите внимание на то, что предположения не выдвигаются наобум. Мы начинаем в середине и движемся вверх или вниз по одному предположению за раз. Поскольку первое предположение значительно выше искомого ответа, мы уменьшаем количество правильных решений на 1 и увеличиваем количество неправильных на 1 за раз, уменьшая сумму на 13 центов.

Задача 10.9

В США существуют монеты следующего достоинства: 1 цент, 5 центов, 10 центов, 25 центов, 50 центов (есть даже монета $1). Какое наименьшее количество монет необходимо, чтобы составить любую сумму от 1 цента до $1?

Обычный подход

Один из подходов — это взять какое-то количество монет каждого достоинства и попытаться найти наименьшее их число, которое позволяет составить любую сумму от 1 цента до $1. Другими словами, реально выполнить необходимые действия. Некоторые пытаются пойти обратным путем и начинают с двух 50-центовых монет. Ни тот ни другой подход нельзя назвать рациональным.

Образцовое решение

Воспользуемся стратегией обоснованного предположения и проверки. Очевидно, что нам понадобятся четыре одноцентовых монеты для получения сумм величиной до 4 центов. Добавив одну пятицентовую монету, мы можем получить любую сумму от 1 цента до 9 центов. Добавление 10-центовой монеты позволяет составить суммы величиной до 19 центов. Еще одна 10-центовая монета делает доступными суммы до 29 центов. Одна 25-центовая монета позволяет составить все суммы до 54 центов. Наконец, одна 50-центовая монета расширяет диапазон доступных сумм до $1. Нам необходимы девять монет следующих достоинств:

1 цент, 1 цент, 1 цент, 1 цент, 5 центов, 10 центов, 10 центов, 25 центов, 50 центов.

Для проверки полученного ответа можно выбрать наугад несколько сумм и попробовать составить их с помощью наших девяти монет. Например, чтобы составить сумму 73 цента, нам потребуются монеты 50 центов, 10 центов, 10 центов и три по 1 центу.

Задача 10.10

Древние египтяне были выдающимися математиками. Пирамиды и многие построенные ими храмы наглядно подтверждают это. Они одними из первых стали пользоваться дробями и представляли их в виде суммы долей единицы. (Доля единицы — это дробь, в числителе которой находится 1.) Так,



Как древние египтяне записали бы дробь?

Обычный подход

Традиционно выписывают различные доли единицы, находят их общий знаменатель и суммируют, чтобы подобрать подходящий набор долей. Ответ практически невозможно получить, если действовать беспорядочно. Количество возможностей здесь почти бесконечно.

Образцовое решение

Простое тыканье наугад редко дает результат. Обоснованное предположение и проверку можно использовать организованно. Проанализируем приведенные выше примеры.

Прежде всего, обратите внимание на то, что все знаменатели долей единицы являются множителями исходного знаменателя. В первом случае знаменатели 2 и 3 являются множителями исходного знаменателя 6. Значит у искомых долей единицы в знаменателе должны стоять множители числа 28. Кроме того заметьте, что доли единицы идут в порядке убывания — впереди стоит наибольшая доля, за ней идет следующая по величине и т. д. Очевидно, что наибольшая доля единицы это Если в качестве возможных знаменателей использовать множители числа 28, то следующей величине долей единицы будет При сложении этих долей мы получим: Нам, однако, нужно еще чтобы получить в сумме Таким образом, искомая сумма единичных долей равна:

Использованный здесь метод подходит для дробей, знаменатель которых представляет собой составное число. Если знаменатель — простое число, то для решения задачи нужна другая процедура.



Оглавление

  • Об авторах
  • Введение
  •   Задача
  •   Решение 1
  •   Решение 2
  •   Решение 3
  •   Решение 4
  •   Решение 5
  •   Решение 6
  •   Решение 7
  • Глава 1 Логическое рассуждение
  •   Задача 1.1
  •     Обычный подход
  •     Образцовое решение
  •   Задача 1.2
  •     Обычный подход
  •     Образцовое решение
  •   Задача 1.3
  •     Обычный подход
  •     Образцовое решение
  •   Задача 1.4
  •     Обычный подход
  •     Образцовое решение
  •   Задача 1.5
  •     Обычный подход
  •     Образцовое решение
  •   Задача 1.6
  •     Обычный подход
  •     Образцовое решение
  •   Задача 1.7
  •     Обычный подход
  •     Образцовое решение
  •   Задача 1.8
  •     Обычный подход
  •     Образцовое решение
  •   Задача 1.9
  •     Обычный подход
  •     Образцовое решение
  •   Задача 1.10
  •     Обычный подход
  •     Образцовое решение
  •   Задача 1.11
  •     Обычный подход
  •     Образцовое решение
  •   Задача 1.12
  •     Обычный подход
  •     Образцовое решение
  • Глава 2 Распознавание закономерности
  •   Задача 2.1
  •     Обычный подход
  •     Образцовое решение
  •   Задача 2.2
  •     Обычный подход
  •     Образцовое решение
  •   Задача 2.3
  •     Обычный подход
  •     Образцовое решение
  •   Задача 2.4
  •     Обычный подход
  •     Образцовое решение
  •   Задача 2.5
  •     Обычный подход
  •     Образцовое решение
  •   Задача 2.6
  •     Обычный подход
  •     Образцовое решение
  •   Задача 2.7
  •     Обычный подход
  •     Образцовое решение
  •   Задача 2.8
  •     Обычный подход
  •     Образцовое решение
  •   Задача 2.9
  •     Обычный подход
  •     Образцовое решение
  • Глава 3 Действие от обратного
  •   Задача 3.1
  •     Обычный подход
  •     Образцовое решение
  •   Задача 3.2
  •     Обычный подход
  •     Образцовое решение
  •   Задача 3.3
  •     Обычный подход
  •     Образцовое решение
  •   Задача 3.4
  •     Обычный подход
  •     Образцовое решение
  •   Задача 3.5
  •     Обычный подход
  •     Образцовое решение
  •   Задача 3.6
  •     Обычный подход
  •     Образцовое решение
  •   Задача 3.7
  •     Обычный подход
  •     Образцовое решение
  •   Задача 3.8
  •     Обычный подход
  •     Образцовое решение
  •   Задача 3.9
  •     Обычный подход
  •     Образцовое решение
  •   Задача 3.10
  •     Обычный подход
  •     Образцовое решение
  • Глава 4 Принятие другой точки зрения
  •   Задача 4.1
  •     Обычный подход
  •     Образцовое решение
  •   Задача 4.2
  •     Обычный подход
  •     Образцовое решение
  •   Задача 4.3
  •     Обычный подход
  •     Образцовое решение
  •   Задача 4.4
  •     Обычный подход
  •     Образцовое решение
  •   Задача 4.5
  •     Обычный подход
  •     Образцовое решение
  •   Задача 4.6
  •     Обычный подход
  •     Образцовое решение
  •   Задача 4.7
  •     Обычный подход
  •     Образцовое решение
  •   Задача 4.8
  •     Обычный подход
  •     Образцовое решение
  •   Задача 4.9
  •     Обычный подход
  •     Образцовое решение
  •   Задача 4.10
  •     Обычный подход
  •     Образцовое решение
  •   Задача 4.11
  •     Обычный подход
  •     Образцовое решение
  •   Задача 4.12
  •     Обычный подход
  •     Образцовое решение
  •   Задача 4.13
  •     Обычный подход
  •     Образцовое решение
  •   Задача 4.14
  •     Обычный подход
  •     Образцовое решение
  •   Задача 4.15
  •     Обычный подход
  •     Образцовое решение
  •   Задача 4.16
  •     Обычный подход
  •     Образцовое решение
  •   Задача 4.17
  •     Обычный подход
  •     Образцовое решение
  •   Задача 4.18
  •     Обычный подход
  •     Образцовое решение
  • Глава 5 Анализ экстремальных ситуаций
  •   Задача 5.1
  •     Обычный подход
  •     Образцовое решение
  •   Задача 5.2
  •     Обычный подход
  •     Образцовое решение
  •   Задача 5.3
  •     Обычный подход
  •     Образцовое решение
  •   Задача 5.4
  •     Обычный подход
  •     Образцовое решение
  •   Задача 5.5
  •     Обычный подход
  •     Образцовое решение
  •   Задача 5.6
  •     Обычный подход
  •     Образцовое решение
  •   Задача 5.7
  •     Обычный подход
  •     Образцовое решение
  •   Задача 5.8
  •     Обычный подход
  •     Образцовое решение
  •   Задача 5.9
  •     Обычный подход
  •     Образцовое решение
  •   Задача 5.10
  •     Обычный подход
  •     Образцовое решение
  •   Задача 5.11
  •     Обычный подход
  •     Образцовое решение
  • Глава 6 Решение более простой аналогичной задачи
  •   Задача 6.1
  •     Обычный подход
  •     Образцовое решение
  •   Задача 6.2
  •     Обычный подход
  •     Образцовое решение
  •   Задача 6.3
  •     Обычный подход
  •     Образцовое решение
  •   Задача 6.4
  •     Обычный подход
  •     Образцовое решение
  •   Задача 6.5
  •     Обычный подход
  •     Образцовое решение
  •   Задача 6.6
  •     Обычный подход
  •     Образцовое решение
  •   Задача 6.7
  •     Обычный подход
  •     Образцовое решение
  •   Задача 6.8
  •     Обычный подход
  •     Образцовое решение
  •   Задача 6.9
  •     Обычный подход
  •     Образцовое решение
  • Глава 7 Организация данных
  •   Задача 7.1
  •     Обычный подход
  •     Образцовое решение
  •   Задача 7.2
  •     Обычный подход
  •     Образцовое решение
  •   Задача 7.3
  •     Обычный подход
  •     Образцовое решение
  •   Задача 7.4
  •     Обычный подход
  •     Образцовое решение
  •   Задача 7.5
  •     Обычный подход
  •     Образцовое решение
  •   Задача 7.6
  •     Обычный подход
  •     Образцовое решение
  •   Задача 7.7
  •     Обычный подход
  •     Образцовое решение
  •   Задача 7.8
  •     Обычный подход
  •     Образцовое решение
  •   Задача 7.9
  •     Обычный подход
  •     Образцовое решение
  •   Задача 7.10
  •     Обычный подход
  •     Образцовое решение
  •   Задача 7.11
  •     Обычный подход
  •     Образцовое решение
  •   Задача 7.12
  •     Обычный подход
  •     Образцовое решение
  • Глава 8 Схематичное изображение, или Визуальное представление
  •   Задача 8.1
  •     Обычный подход
  •     Образцовое решение
  •   Задача 8.2
  •     Обычный подход
  •     Образцовое решение
  •   Задача 8.3
  •     Обычный подход
  •     Образцовое решение
  •   Задача 8.4
  •     Обычный подход
  •     Образцовое решение
  •   Задача 8.5
  •     Обычный подход
  •     Образцовое решение
  •   Задача 8.6
  •     Обычный подход
  •     Образцовое решение
  •   Задача 8.7
  •     Обычный подход
  •     Образцовое решение
  •   Задача 8.8
  •     Обычный подход
  •     Образцовое решение
  •   Задача 8.9
  •     Обычный подход
  •     Образцовое решение
  • Глава 9 Учет всех возможностей
  •   Задача 9.1
  •     Обычный подход
  •     Образцовое решение
  •   Задача 9.2
  •     Обычный подход
  •     Образцовое решение
  •   Задача 9.3
  •     Обычный подход
  •     Образцовое решение
  •   Задача 9.4
  •     Обычный подход
  •     Образцовое решение
  •   Задача 9.5
  •     Обычный подход
  •     Образцовое решение
  •   Задача 9.6
  •     Обычный подход
  •     Образцовое решение
  •   Задача 9.7
  •     Обычный подход
  •     Образцовое решение
  •   Задача 9.8
  •     Обычный подход
  •     Образцовое решение
  •   Задача 9.9
  •     Обычный подход
  •     Образцовое решение
  • Глава 10 Обоснованное предположение и проверка
  •   Задача 10.1
  •     Обычный подход
  •     Образцовое решение
  •   Задача 10.2
  •     Обычный подход
  •     Образцовое решение
  •   Задача 10.3
  •     Обычный подход
  •     Образцовое решение
  •   Задача 10.4
  •     Обычный подход
  •     Образцовое решение
  •   Задача 10.5
  •     Обычный подход
  •     Образцовое решение
  •   Задача 10.6
  •     Обычный подход
  •     Образцовое решение
  •   Задача 10.7
  •     Обычный подход
  •     Образцовое решение
  •   Задача 10.8
  •     Обычный подход
  •     Образцовое решение
  •   Задача 10.9
  •     Обычный подход
  •     Образцовое решение
  •   Задача 10.10
  •     Обычный подход
  •     Образцовое решение

  • Наш сайт является помещением библиотеки. На основании Федерального закона Российской федерации "Об авторском и смежных правах" (в ред. Федеральных законов от 19.07.1995 N 110-ФЗ, от 20.07.2004 N 72-ФЗ) копирование, сохранение на жестком диске или иной способ сохранения произведений размещенных на данной библиотеке категорически запрешен. Все материалы представлены исключительно в ознакомительных целях.

    Copyright © читать книги бесплатно